You are on page 1of 36

OBSTETRICS & GYNECOLOGY 1.

Which of the following is the correct flow of blood from the uterine wall to the endometrium?

A. B. C. D.
2.

Uterine artery arcuate artery radial artery straight & coiled spiral artery Uterine artery radial artery arcuate artery straight & coiled spiral artery Uterine artery arcuate artery straight artery radial & coiled spiral artery Uterine artery straight artery arcuate artery radial & coiled spiral artery

What is the functional life span of the corpus luteum?

A. B. C. D.
3.

7 + 2 days 14 + 2 days 21 + 2 days 28 + 2 days

What hormone is secreted by the dominant ovarian follicle? A. estriol

B.
C. D. 4.

estrone estradiol progesterone

During the embryonic period, where is the formation of blood first demonstrable? A. bone marrow B. yolk sac C. liver D. spllen During the secretory phase, what is the uppermost layer from the uterine cavity ? A. Zona compacta B. Zona basalis C. Zona spongiosa D. Decidua basalis What stage of human development is implanted in the uterine cavity? A. blastomeres B. embryo blastocyst D. morula How many new primary oocytes are there during puberty? A. 0 B. 1,000 C. 10,000 D. 100,000 What is the important event that occurs prior to implantation? A. Formation of daughter cells B. Extrusion of the polar body C. Accumulation of fluid between blastomeres D. Disappearance of the zona pellucida At what phase does regresson of the corpus luteum occur? A. At the end of the proliferative phase B. At the end of the secretory phase C. During menstruation D. After ovulation A 34 year old G4P4 delivered vaginally an 8 lb baby at home assisted by a hilot. The placenta was delivered without difficulty. However, a few minutes later, there was profuse vaginal bleeding and the patient wa rushed to the hospital. In the ER, the patient was hypotensive, tachycardic and pale. On abdominal examination, the uterine fundus was soft and above the umbilicus. There were no vaginal or cervical lacerations. What is the most probable diagnosis?

5.

6.

C.

7.

8.

9.

10.

A. B. C.
D. 11.

uterine inversion retained placental fragments uterine rupture uterine atony

A 33 year old G3P2 PU 18 weeks consulted at the ER because of watery vaginal discharge accompanied by hypogastric pain. Vital signs were normal. Speculum exam revealed pooling of watery discharge. I.E. revealed an open cervix, palpable fetal parts at the os, uterus enlarged to 18 weeks AOG. What is the most probable diagnosis? A. Recurrent abortion

B.
C. D. 12.

Incomplete abortion Inevitable abortion Threatened abortion

At what part of the fallopian tube does tubal rupture occur earliest? A. Interstitial

B.
C. D. 13.

B. Ampullary Isthmic Fimbria

What is the most commonly associated condition for abruption placenta? A. External trauma

B.
C. D. 14.

Pregnancy- induced hypertension alcohol consumption Short cord

Which of the following transvaginal utrasonographic cervical findings correlate positively with preterm delivery?

A.

negative transfundal pressure funneling 2.7 cm cervical length T- shaped cervix

B. C.
D. 15.

Preterm infant is an infant who is: A. less than 2000 grams at birth less than 2500 grams at birth C. less than 37 weeks AOG D. less than 38 weeks AOG A 35 year old G1P0 had an infertility work-up fro which she was prescribed clomiphene citrate. She got pregnant and was diagnosed to have twin pregnancy. What is the most probable type of twinning? A. Monozygotic

B.

16.

B.
C. D. 17.

Dizygotic Conjoined Locked

Which of the following is the most important parameter in the assessment of patient in true labor? A. intactness of the amniotic membrane B. cervical dilatation and effacement C. presenting part D. bony pelvis The level of the presenting part in the birth canal described in relationship to the ischial spines, which is halfway between the pelvic inlet and the pelvic outlet is called A. position B. B. effacement C. Dilatation D. station The characteristic curve pattern of cervical dilatation in a normal labor is described as: A. A. hyperbolic B. sigmoidal C. diagonal straight D. horizontal A 30 year old G1P0, term was admitted for labor pains. FH- 34 cm, FHT- 140 bpm. IE- cervix is 4 cm dilated, 60% effaced, cephalic, station -2, rupture BOW. Uterine contractions every 2-3 mins, moderate. After 2 hours, IEcervix 4-5 cm dilated, 70 % effaced, station -2. After 2 hours, IE- cervix is 5-6 cm dilated, 80% effaced, station -1.Describe the progress of labor. A. Normally progressing

18.

19.

20.

B. C.
D. 21.

Protracted cervical dilatation Protracted descent Arrest in descent

What phase of the active labor reflects the feto-pelvic relationship? A. latent phase B. B. acceleration phase C. phase of maximum slope D. deceleration phase A 19 year old G1P0 PU 40 weeks, not in labor, was seen at the OPD for decreased fetal movement. She was hooked to an electronic fetal monitor and tracing showed: Baseline FHT- 140s, good variability, with more than 2 accelerations of 20 bpm lasting for 20 secs. The tracing is interpreted as:

22.

A.
B.

reactive non-reactive positive negative

C.
D. 23.

Fetal tachycardia is defined as a baseline heart rate greater than: A. 140 bpm B. 150 bpm C. 160 bpm D. 170 bpm What is the presentation type when the fetal head is artially flexed with the anterior fontanel or bregma is presenting? A. face B. vertex C. brow D. sinciput You were the intern on duty in the ER and you did an abdominal exam on a 22 year old G2P1 PU 37 weeks who consulted because of hypogastic pain. You found out that the fundus is occupied by a hard ballotable mass. What Leopolds maneuver did you perform?

24.

25.

A.
B. C. D. 26.

LM 1 LM 2 LM 3 LM 4

There is an increase in the size of cardiac silhouette in X-ray during pregnancy because the heart is displaced to the: A. left and upward B. left and downward C. right and dowmward D. right and upward

27.

During pregnancy, the diaphragm rises by

A.
B. C. D. 28.

2 cm 4 cm 6 cm 8 cm

Impaired gall bladder contraction during pregnancy is due to A. estrogen

B.
C. D. 29.

estrogen and progesterone progesterone anatomical change in gall bladder

Naegeles rule is use to estimate the expected date of delivery by ____. A. adding 3 days to the first day of PMP and count back 7 months B. adding 7 days to the first day of PMP and count back 3 months C. adding 7 days to the first day of bleeding and count back 3 months D. adding 7 days to the first day of LMP and count back 3 months Which of the following is proven teratogen? A. Vitamin A derivatives

30.

B.
C. D. 31.

Metronidazole Cephalosporins Ampicillin

Which of the following vaccines is contraindicated during pregnancy? A. Pneumococus

B.
C. D. 32.

Hepatitis B Influenza Mumps, measles, rubella

The preferred method for the delivery of the aftercoming head is A. Pipers forceps extraction B. Mauriceau-Smellie-Veit Manuever C. Bracht maneuver

D.
33.

Prague maneuver

You were assigned to deliver the baby of a 25 year old G1P0 PU 38 weeks. You applied the forceps on the fetal head with the following findings: head was at station +2 with the sagittal suture at left occiput anterior position. This is classified as A. outlet forceps delivery B. low forceps delivery C. midforceps delivery D. high forceps delivery A 22 year old G1P0 patient at 39 weeks AOG was admitted for elective Cesarean Section for breech presentation. She requested to her obstetrician that a transverse suprapubic abdominal incision be done to her. This type of incision is called A. Kerr B. Kronig

34.

C.
D. 35.

Pfannensteil Classical

Which of the following is one of the requirements that must be present before obstetric forceps must be used? A. The membranes should be intact B. Cephalic presentation C. The fetal head must be floating D. The cervix must be fully dilated and retracted The single most significant risk factor in the development of post-partum pelvic infection is A. early rupture of membranes B. Prolonged labor C. Cesarean delivery D. Multiparity The process by which the uterus returns to its normal size, tone and position after delivery is called A. involution

36.

37.

B. C.
D. 38.

puerperium subinvolution atony

What is the mechanism behind the increase in cardiac output right after delivery? A. maternal exhaustion B. increase caval compression C. sympathetic stimulation D. autotransfusion Congenital rubella syndrome is more likely common during which AOG? A. 8-10 weeks B. 12-14 weeks C. 16-18 weeks D. 36-38 weeks What is the diagnostic procedure of choice for identifying gallstones in pregnancy? A. CT Scan B. X-Ray C. Ultrasound D. MRI

39.

40.

41.

A patient consulted because her husband is a seaman and will be coming home in 2 months for a 1-month vacation. She just had her menses 2 days ago. What is the most effective reversible form of contraception will you give? A. combined oral contraceptive B. calendar rhythm method C. Depo-Provera D. combined oral contraceptives A 30 year old G1P1 consulted at the OPD for Pap smear. According to her, she had an IUD in-situ for 1 year. On PE, you can not visualize the tail of the IUD string. What is the best thing to do for this patient? A. Assume that the device has been expelled B. Assume that the patient is telling a lie C. Perform an transvaginal ultrasound D. Assume that the device has been expelled and perform the Pap smear A 35 y.o., G3P3 (3-0-0-3) complained of scanty menstrual flow and continuous severe cramping throughout the menstrual period after undergoing cryotherapy due to chronic cervicitis. What is the most likely cause of her complaint? A. pelvic inflammation B. ectopic endometrial tissue C. cervical stenosis D. stress and tension A 21 y.o. patient, nulligravid , single came in because of severe vaginal bleeding of 2 days duration. What is the management of choice in this case? A. D & C B. High dose progestins C. High dose estrogen D. Hysteroscopy The most common cause of DUB in the premenarcheal and postmenopausal woman is _____. A. Ovulatory B. Anovulartory C. Organic D. Iatrogenic A 32 y.o., G2P2 (2-0-0-2) consuted for amenorrhea since delivery up to almost 1 year after. Breastfeeding was not practiced.The last pregnancy was delivered via NSD with history of uterine atony and blood transfusion. What is the most likely cause of her amenorrhea? A. Ashermans syndrome B. Sheehans syndrome C. Simmonds syndrome D. Polycystic ovarian syndrome Menometrorrhagia is defined as: A. Abnormal uterine bleeding occurring at regular intervals B. Prolonged uterine bleeding at irregular intervals C. Normal amount of vaginal bleeding at frequent intervals D. Decreased amount of vaginal bleeding at frequent intervals Which of the following statements is true of DUB? A. Anovulatory bleeding is the most common cause in the premenarcheal years B. There is continuous estrogen production without corpus luteum formation C. Halbans syndrome is a common cause of DUB D. It is usually associated with severe dysmenorrhea The most common histologic type of vaginal cancer is:

42.

43.

44.

45.

46.

47.

48.

49.

A. B.
C. D.

Squamous carcinoma Adenocarcinoma Malignant melanoma Sarcoma

50.

A 69- year old G4P4 (4004) consulted for an ulcerated mass on the right labium majus. She had been to several physicians who have prescribed unrecalled topical creams and solutions without relief. Upon seeing the patient, your recommendation would be: A. Observation B. Steroid topical cream C. Excision biopsy of the mass D. Simple vulvectomy The area of the cervix that is most prone to precancerous and cancerous neoplasms is the: A. Histologic portio

51.

B.
C. D. 52.

Transformation zone Histologic endocervix Isthmus

Which of the following HPV Types is associated with high oncogenic potential? A. HPV Type 1

B.
C. D.

HPV Type 5 HPV Type 6 HPV Type 18

53. A 37 year old, G3P3 was admitted because of vaginal bleeding. Pregnancy test is negative. She underwent diagnostic curettage. While doing the curettage humps and bumps were noted. What is the most likely diagnosis? A. intramural myoma B. subserous myoma C. submucous myoma D. broad ligament myoma

54.

A 68 year old woman has a biopsy result of atypical complex hyperplasia. What is the most appropriate treatment for her? A. Judicious observation B. Repeat fractional D&C after 6 months C. Give cyclic progestin therapy to promote monthly withdrawal bleeding D. Perform TAHBSO This granulose-theca cell A. B. C. D. tumor has this characteristic inclusion body. Psamomma bodies Call-Exner bodies Schiller Duvall bodies keratin pearls

55.

56. A 33 year old nulligravid patient consulted in your clinic because of cervical mass. She was diagnosed as a case of prolapsed myoma . What is the best management for her case? A. Do myomectomy by laparotomy B. Do subtotal hysterectomy C. Do transcervical resection of the myoma D. Do total abdominal hysterectomy only 57. A 28 year old, single, nulligravid patient consulted in the OPD for the result of her pelvic ultrasound. It revealed a 18mm x 10mm x 14mm and a 13mm x 16mm x 10 mm intramural myomatas. She is asymptomatic. 58. A. B. C. D. 59. How will you manage the patient? Work up the patient , then schedule her for myomectomy Give her GnRH agonists Reevaluate the patient at 6-month interval to determine the rate of growth Perform fractional D & C This is the diagnostic procedure of choice for endometrial cancer. A. Progesterone challenge test B. Ultrasound C. Pap smear D. Fractional D&C This is the drug of choice for syphilis. Oral doxycycline Oral tetracycline Oral Penicillin Parenteral Penicillin G

60. A. B. C. D.

61. A 29 year old sexually active female consulted in your clinic because of vulvar lesion. History revealed that it started as paresthesia of the vulvar skin then papule and subsequent vesicle formation. Simple clinical inspection revealed ulcers which are painful when touched with cotton-tipped applicator. The most likely diagnosis is _____. A. Syphilis

B.
C. D. 62. A. B. C. D. 63.

Granuloma inguinale LGV Genital herpes What are the most important goals of the medical therapy of acute PID? prevention and treatment of the disease early diagnosis and prompt treatment of the disease prevention of the disease and preservation of tubal function resolution of symptoms and preservation of tubal function This occurs when increased intraabdominal pressure is not transmitted equally to the bladder and the functional urethra. A. detrussor instability

B.
C. D. 64.

urethral sphicteric dysfunction genuine stress incontinence true incontinence

This occurs when a bladder is overdistended because of its instability to empty. A. true incontinence

B.
C. D. 65.

overflow incontinence genuine stress incontinence detrussor instability

A 35 year old G1P0 (0-0-1-0) consulted because failure to become pregnant 1 year after her abortion. Her condition is considered A. unexplained infertility B. primary infertily C. secondary infertility D. normal after an abortion The first drug to offer in women with anovulation is A. bromocriptine B. GnRH C. Gonadotrophins D. clomiphene citrate The most common cause of tubal/peritoneal factors of infertility is A. surgery on the tubes B. tuberculosis C. PID

66.

67.

D.
68.

endometriosis

Among the factors causing female infertility, the easiest to diagnose and manage is A. cervical factors B. uterine factors

C.
D. 69.

ovulatory factors tubal/peritoneal factors

An absolute contraindication to hormone replacement therapy is:

A.
B.

Thromboembolic disease Bronchial asthma Diabetes mellitus Hypertension

C. D.
70.

Over the counter pregnancy test kits will test for which placental hormone? ((Baja-Panlilio Chapter 6, p. 64) A. estrogen B. progesterone C. human placental lactogen D. human chorionic gonadotropin Mefenamic acid taken by pregnant women may result in the closure of what structure? (Baja-Panlilio, Vol I pp. 8283) A. ductus venosus B. foramen ovale C. ductus arteriosus D. ventricular septal defect Hydroureter during pregnancy is more marked on the right than on the left because of: (Baja panlilio Chapter 8 Page 107) A. renal artery crossing the ureter on the right B. dextrorotation of the uterus C. sigmoid colon on the right D. majority of fetus staying on the right maternal side In which of the following are relatively low levels of hCG detected in maternal blood? (Baja Panlilio Chapter 6 page 64) A. Down syndrome B. hydatidiform mole C. multiple gestation D. Impending abortion The majority of spontaneous abortions are due to: (Baja-Panlilio, Chapter 27, p. 317) A. chronic infections B. endocrine abnormality C. chromosomal defects D. uterine synechial A 39 year old, G4P3 (3003) patient with a history of repeated episodes of pelvic inflammatory disease was diagnosed to have an unruptured ectopic pregnancy. The cause of her ectopic pregnancy is: (Baja-Panlilio, Chapter 28, p. 327) A. uterine tumor B. adhesions C. ovarian tumor D. salpingitis The best basis for the diagnosis of Preterm Labor in this patient is the presence of: (Baja Panlilio Chapter 37 , page 355) A. mucoid vaginal discharge B. painful uterine contractions C. contractions occurring every 10 to 15 minutes D. cervical dilatation and effacement A blood pressure of 160/110 mmhg. Proteinuria of 4 gm/day, with elevated liver enzymes is classified as: (BajaPanlilio, Chapter 35 p 231) A. preeclampsia mild B. preeclampsia severe C. chronic hypertension D. gestational hypertension A 29 year old G1P0, consulted for the 1st time on her 28 weeks AOG. Her BP was 160/120 mmHg. She had severe headache and her fundic height was only at the level of the umbilicus. The appropriate laboratory exams to be done initially is: (Baja-Panlilio, Chapter 35 p. 337 A. non-stress test B. doppler velocimetry C. biophysical profile D. liver enzymes A 35 year old, G6P5 (5005) on her 12th week of pregnancy was diagnosed on ultrasound to have an h-mole. What will be the management for this case? ((Baja-Panlilio, Chapter 30, p. 350) A. single agent chemotherapy B. hysterotomy C. suction curettage D. hysterectomy, followed by prophylactic chemotherapy When the long axis of the fetus parallels the longitudinal axis of the uterus, the lie of the fetus is called: (Panlilio, Textbook of Obstetrics (Pathologic and Physiologic) 2nd ed, Page 210) transverse longitudinal

71.

72.

73.

74.

75.

76.

77.

78.

79.

80.
A.

B.
C. cephalic D. breech 81.

The tracings showed that the fetus: (Baja-Panlilio, Chapter 21 pp. 237

A. B. C. D. 82.

has normal tracings has a late deceleration has an early deceleration has a variable deceleration

In intrapartum monitoring, the management for severe bradycardia preceded by late deceleration and absent variability is done by: (Baja-Panlilio, Chapter 21 pp. 236 A. giving oxygen inhalation at 3-4 L/mins. B. infusion of intravenous fluids C. immediate termination of pregnancy D. placing patient at left lateral decubitus A 25 year old G1P0, 38-39 wks. AOG consulted at the emergency room due to labor pains. She has no prenatal check-ups and family history revealed diabetes mellitus in sister and mother. Abdominal exam revealed fundic height = 40 cm, uterine contractions every 2-3 mins, 45-50 secs. duration. Internal exam has remained unchanged at 6 cms dilated, fully effaced, station -2, cephalic, (-) BOW for the past 2 hrs. The serious complication during vaginal delivery of this baby where there is arrest in delivery of the shoulder is: (BajaPanlilio, . 421-422) A. Shoulder dystocia B. Deep transverse of the head C. Erbs palsy D. Prolonged second stage of labor A well-nourished patient on her third trimester of pregnancy has a Hemoglobin value of 10.5 gms/dl. This low value could be explained by: (Baja Panlilio 2nd Ed Chapter 8 page 112) A. iron deficiency anemia B. increase in blood volume C. no increase in RBC production D. bone marrow failure during pregnancy Iron supplementation during pregnancy is mandatory because of: (Baja Panlilio 2nd Ed Chapter 8 page 112) A. increased physiologic Fe loss during pregnancy B. poor Fe absorption during pregnancy C. increased demand by the increased production of RBCs D. poor bone marrow response to anemia Pregnancy is said to be a diabetogenic state because of: (Baja Panlilio 2nd Ed Chapter 8 page 114) A. decreased insulin production B. increased caloric intake of the mother C. increased fat utilization D. increased insulin resistance An increase in the following hormone is an indication of Thyrotoxicosis during pregnancy: (Baja Panlilio 2nd Ed Chapter 54 page 595 A. free Thyroxine hormone B. total Thyroxine hormone C. thyroid Stimulating Hormone D. thyroid Releasing Hormone A puerpera came for her postnatal follow up 2 weeks after an uncomplicated vaginal delivery. The following are expected findings on her physical examination: (Panlilio, Textbook of Obstetrics (Pathologic and Physiologic) 2nd ed, Pages 295-298) A. uterus at the level of the symphysis pubis B. lochia alba C. cervix open and thick D. bipedal edema A week after delivery by emergency cesarean section after a prolonged labor, a patient came because of fever of 38 C. Puerperal infection is suspected if she has: (Panlilio, Textbook of Obstetrics (Pathologic and Physiologic) 2nd ed, Pages pages 556-561) A. breast engorgement B. foul smelling lochia and tender uterus C. milk fever D. thrombophlebitis A 42 year old G2P1 at her 32 weeks gestation with known renal disease and hypertension presents with BP of 220/120 mmHg but is asymptomatic. The diagnostic test you will perform to detect chronicity of her illness is: (Baja-Panlilio, Chapter 35 p. 342) A. Doppler velocimetry

83.

84.

85.

86.

87.

88.

89.

90.

B. C. D. 91.

urine protein fundoscopy elevated serum creatinine

The monitoring done to detect increased severity of preeclampsia is: (Baja-Panlilio, Chapter 35, p. 336) A. maternal fibronectin B. urine protein C. serum uric acid D. serum creatinine The most common pathophysiologic mechanism in perimenopausal bleeding is: (Compre Gyne, 1082) A. cervical ancer B. endometrial cancer C. anovulation D. abnormal pregnancy states A 34 yo G3P3 (3-0-0-3) presents with episodes of missed period for 2 cycles then irregular and profuse bleeding for the past two weeks. Her pregnancy test is negative with unremarkable pelvic exam findings except for the moderate bleeding. What is the most likely diagnosis: (Compre Gyne, p. 1082-1083) A. threatened abortion B. hydatidiform mole C. dysfunctional uterine bleeding D. endometrial cancer A 60 yo G5P5 (5-0-0-5) has been menopausal for the past 12 years presents with minimal vaginal bleeding. What diagnostic exam will you recommend? (Compre Gyne, p. 1082-1083) A. colposcopy B. laparoscopy C. diagnostic Dilatation and Curettage D. Transvaginal Ultrasound An adnexal cystic mass was seen appreciated on TVS, 6 cm in diameter, in a 24 yo patient who presents with abnormal vaginal bleeding. What is the most likely diagnosis? (Compre Gyne, p. 506-507) A. follicular cyst B. dermoid cyst C. serous cyst D. corpus luteum cyst A 5 year old child was initially treated with antibiotics for purulent vaginal discharge for a week. On follow up, the discharge was noted to be foul smelling and bloody. The most probable cause is: (Comprehensive Gynecology, 4th ed, page 274-277) A. monilial infection B. child molestation C. foreign body in the vagina D. precocious puberty

92.

93.

94.

95.

96.

97.

A mother is concerned with the appearance of whitish, non puritic vaginal discharge on her 11 year old child noted since 8 month preceding menarche. It is best to; (Comprehensive Gynecology, 4th ed, page 276) A. do gram stain on the discharge B. advise vaginal douche C. prescribe oral antibiotics D. reassure the mother and the child that the discharge is normal The most frequent symptoms of endometrial hyperplasia is: (Compre Gyne, p. 870) A. foul smelling vaginal discharge B. abnormal vaginal bleeding C. pelvic pain D. alternating constipation and diarrhea A 46 yo G4P4 (4-0-0-4) with a nodular uterus, enlarged to 20 weeks AOG presents with menorrhagia. What is the most likely diagnosis? (Comprehensive Gynecology Chapter 18 Page 502) A. Subserous myoma B. Submucous myoma C. Adenomyosis D. Abnormal pregnancy A 65 year old nulligravida consulted at the emergency room due to postmenopausal bleeding for 3 yrs. Shes obese and known to be hypertensive for 10 years. Her menstrual history revealed irregularly irregular cycles. She was treated for breast cancer 6 years ago and has been taking tamoxifen for the past 5 years. The most probable cause of her bleeding is a pathology in the: (Compre Gyne, p. 860-867) A. Cervix B. Endometrium C. Ovary D. Vagina A 35 year old, G6P6 (6006) wife of a seaman, consulted due to postcoital bleeding. Speculum exam revealed a flat warty lesion along the posterior cervical lip. Histopathology of cervical punch biopsy done revealed dysplastic cells involving nearly the whole thickness of the epithelium. The most probable diagnosis is CIN? (Compre Gyne, 802-803) A. I B. II C. III D. IV A 65 year old nulligravida consulted at the emergency room due to postmenopause bleeding x 3 yrs. Shes obese and known to be hypertensive for 10 years. Her menstrual history revealed irregularly irregular cycles. She was treated for breast cancer 6 years ago and has been taking tamoxifen for the past 5 years. The most appropriate diagnostic test is: (Compre Gyne, 870-871) A. Pap smear B. Cervical punch biopsy C. Fractional curettage

98.

99.

100.

101.

102.

D.

Transvaginal ultrasound

103.

A 25 year old, G6P0 (0060) consulted at the emergency room due to postcoital bleeding x 3 months duration. Shes a victim of child prostitution. On pelvic exam, there was a 2 cm cauliflower like lesion on the anterior lip of the cervix. The vagina and parametria are smooth. What is the most appropriate diagnostic test? (Compre Gyne, p. 844) A. pap smear B. colposcopy with biopsy C. direct punch biopsy D. cone biopsy A 20 year old commercial sex worker presented at the clinic due to painful shallow ulcers in the vulva associated with burning sensation during urination. She also has multiple oral ulcers at the time of consultation. The most likely diagnosis is: (Compre Gyne, , Chapter 22, page 656) A. Herpes simplex infection B. granuloma inguinale C. Lymphogranuloma venereum D. Syphilis A 38 year old diabeteic patient presented at the clinic due to vaginal pruritus and whitish vaginal discharge. On examination, the vulva is beefy red in appearance with whitish curdled discharge. The most likely diagnosis is: (Compre Gyne, Chapter 22, page 669) A. Trichomoniasis B. Candidiasis C. Bacterial vaginosis D. Mucopuruloent Cervicitis The presence of heavy concentration of coccobacilli surrounding vaginal epithelial cells with loss of distinct cell margins is the appearance of; (Compre Gyne, Chapter 22, page 671) A. Donovan bodies B. Clue cells C. chancre D. inclusion cells A patient with chronic ulcers in the vulva had smears done taken from the ulcers. Findings shows presence of dark staining bacteria with a bipolar appearance found in the cytoplasm of large mononuclear cells .These are diagnostic of ; (Compre Gyne, Chapter 22, page 660 A. Granuloma inguinale B. Lymphogranuloma venereum C. Chancroid D. Syphilis A patient with painless vulvar ulcers came with a positive screening test for syphilis. Confirmatory test that should be done can either be any of the following except: (Compre Gyne, Chapter 22, page 664) A. RPR B. TPI C. FTA-ABS D. MHA-TP

104.

105.

106.

107.

108.

109.

According to CDC, treatment of patients diagnosed with HIV includes the following except: (Compre Gyne Chapter 22, page 686) A. behavioral B. psychosocial C. emotional D. medical A patient diagnosed with gonorrhea should: (Compre Gyne, Chapter 22, page 692) A. be treated with Chloramphenicol B. also be treated for Chlamydia infection C. have follow-up cultures done for asymptomatic women D. not have serologic test for syphilis if cultures for gonorrhea are positive Linda, 65 year old, G7P7 came to your clinic complaining of vaginal itching with burning discomfort. This condition may be due to a decrease in what hormone? (Compre Gyne, p.1223) A. LH B. progesterone C. estrogen D. FSH Linda, 18 year old, delivered an 8 lbs baby. There was note of laceration on the lateral wall off the vaginal vault with profuse vaginal bleeding. There was a sudden drop of BP. Post partum there was note of amenorrhea. Lab examination shows destruction of the pituitary gland. Linda has what syndrome? (Compre Gyne p. 1116) A. Simmonds syndrome B. Sheehans syndrome C. Edwards syndrome D. Ashermans syndrome Factors promoting puerpueral infection include: A. prolonged rupture of membranes B. limited number of vaginal examination C. normal hemoglobin levels D. normal labor Textbook of Obstetrics (Baja-Panlilio, et al), p. 517-8

110.

111.

112.

113.

114.

The fetal heart tones can be best heard in this area if the following were the Leopolds findings: L1- large nodular mass L2 hard, resistant structure at the right side of the mother Small, irregular, mobile parts on the left side of the mother L3 movable hard round mass L4 tips of the fingers able to meet A. right lower quadrant B. left lower quadrant C. right upper quadrant D. left upper quadrant Textbook of Obstetrics (Baja-Panlilio, et al), p. 122-123

115.

The patient with abnormal uterine bleeding is a: A. 18 y.o. whose interval of menses is 24 to 30 days B. 41 y.o. whose menses last 8 to 10 days C. 29 y.o. on DMPA with occasional vaginal spotting D. 22 y.o. leukemic patient with menstrual blood loss of 80 ml Compre Gyne 4th ed, p. 1079-80 A 55 y.o. G5P5 (5005) consulted for fish-wash like vaginal discharge and on-and-off vaginal bleeding. Pelvic exam showed the cervix to be converted to a 6 x 5 cm nodular, fungating mass extending to the R lateral fornix, the right parametria nodular and fixed while the left was free. Based on the information given, this patient can be clinically staged as A. IIB B. IIIA C. IIIB D. IVA Compre Gyne 4th ed, p. 897

116.

117.

A 53 y.o. G1P1 (1001) underwent exploratory laparotomy for an ovarian new growth. Intraoperative findings showed the right ovary to be converted to a 10 cm predominantly cystic mass with excrescences on its outer capsule. The left ovary was grossly normal. All other abdominopelvic organs were grossly normal. Based on the information given, the Intraoperative stage of this patient is A. IA B. IB C. IC D. IIA Compre Gyne 4th ed, p. 966

118.

Speculum exam of a 27 y.o. complaining of leucorrhea showed copious frothy greenish vaginal discharge with strawberry-like mucosa. This is most likely due to: A. candidiasis B. trichomoniasis C. gonococcal infection D. bacterial vaginosis Compre Gyne 4th ed, p. 672-73

119.

The main arterial blood supply of the uterus is a branch of : A. pudendal artery B. external iliac C. ovarian artery D. hypogastric artery The uterus is derived from the: A. Wollfian duct B. Gartners duct C. Mullerian duct D. Urogenital sinus

120.

121.

The violaceous discoloration of the vagina during pregnancy is called: A. Godells sign B. Hegars sign C. Chadwicks sign D. Pawliks sign Which of the following is markedly increased by about 28 weeks gestation? A. fetal movement B. plasma volume C. amniotic fluid D. human chorionic gonadotropin This maneuver is done to promote extension of the fetal head: A. Woods maneuver B. Mauriceaus maneuver C. Ritgens maneuver D. Roberts maneuver One of the following is a presumptive sign of pregnancy: A. softening of the isthmus B. outlining of the fetus C. violaceous vaginal mucosa D. ballottement The fundus of the uterus is at this level at 12 weeks gestation:

122.

123.

124.

125.

10

A. B. C. D. 126.

Just below the umbilicus Above the symphysis At the level of the symphysis Midway between the symphysis and the umbilicus

During the second and third trimester, daily caloric intake should be increased by: A. 400 B. 300 C. 200 D. 100 Lightening is the result of : A. dilatation of the cervix B. descent of the fetus C. increase in Braxton Hicks contractions D. expulsion of bloody show The relation of the point of direction to the right and left of the maternal birth canal is called: A. B. C. D. presentation posture position station

127.

128.

129.

Which ligament is considered as the strongest support of the uterus? A. Cardinal B. broad C. utero-sacral D. round Average duration of the first stage of labor in primigravidas: A. 24 hours B. 12 hours C. 8 hours D. 5 hours Average duration of the third stage of labor among multiparous patients: A. 5 minutes B. 20 minutes C. One hour D. Two hours Milk ejection is the result of the action of: A. Oxytocin B. prolactin C. estrogen D. human placental lactogen Complete anesthesia for abdominal delivery necessitates a block from: A. T10 to S5 B. T10 to S1 C. T8 to S1 D. T8 to S5 The motor pathways to the uterus leaves the spinal cord at the level of: A. T9T10 B. T7T8 C. T6 D. T7 Phase 0 of parturition is characterized by: A. uterine tranquility B. ripening of the cervix C. development of the lower uterine segment D. progesterone withdrawal Secondary arrest of cervical dilatation is cessation of cervical dilatation for: A. one hour or more B. two hours or more C. three hours or more D. 12 hours or more TRUE of hypertonic uterine contractions EXCEPT:

130.

131.

132.

133.

134.

135.

136.

137.

A.
B. C. D. 138.

absence of basal hypertonus usually respond to sedation distorted gradient pressure absence of fundal dominance

Method of delivery in a 19 year old primigravid patient, 39 weeks pregnant, transverse lie in labor:

A.
B. C. D.

internal podalic version under general anesthesia emergency low transverse cesarian section emergency classical cesarian section internal podalic version with complete breech extraction

139.

Significant oligohydramnios is defined as an amniotic fluid index of ____ cm. or less:

A.
B. C.

20 15 10

11

D. 140.

Patient with heart disease without any obstetrical indication are best delivered by:

A.
B. C. D. 141.

cesarian section under general anesthesia normal spontaneous delivery under pudendal block outlet forceps extraction under epidural anesthesia cesarian section under epidural anesthesia

Which antihypertensive is NOT recommended during pregnancy?

A.
B. C. D. 142.

methyl- dopa ace inhibitors hydralazine nifedipene

The following are beta agonist tocolytic agents EXCEPT: A. Ritrodrine B. salbutamol C. indomethacin D. terbutaline Which of the following is NOT true in the use of corticosteroids in premature labor?

143.

A.
B. C. D. 144.

delivery is best delayed 24 hours after the last dose of the drug betamethasone is given at a dose of 12 mg. im every 24 hours X 2 doses it produces induction of fat cells that regulate fetal lunf maturity it affects biochemical systems within type II cells that produce surfactants

Complete expulsion of sperm stored in the reproductive tracr beyond the interrupted vas deferens takes about _____ ejaculations: A. 2 B. 10 C. 20 D. 30 A form of gestational trophoblastic disease characterized by excessive trophoblastic proliferation and edema of the villous stroma without excessive local invasion is: A. H. mole B. Invasive mole C. Choriocarcinoma D. Placental site trophoblastic tumor Internal examination in cases of abruption placenta maybe done to determine: A. location of placenta B. if bag of water has ruptured C. cervical dilatation D. All of the above In uterine atony, the source of bleeding is the: A. uterine lacerations B. placental implantation site C. cervical lacerations D. myometrium What maneuver in shoulder dystocia involves flexing the maternal thighs on the abdomen? A. B. C. D. Woods Pinard Zavanelli McRoberts

145.

146.

147.

148.

149.

A woman using the oral contraceptive pills for the first time should be advised to start taking it : A. on day 1 of menses B. on day 5 of menses C. on day 7 of menses D. anytime as long as she is not pregnant Management of choice for procidentia uteri in a 60 year old multiparous patient with marked cystorectocoele: A. observation and close follow up B. vaginal hysterectomy C. vaginal hysterectomy with anterior-posterior repair D. total abdominal hysterectomy with anterior-posterior repair The following are characteristics of Rokitansky syndrome EXCEPT: A. phenotypically female B. normal ovaries C. normal uterus D. absent vagina The following are congenital anomalies of the mullerian duct EXCEPT: A. imperforate hymen B. transverse vaginal septum C. unicornuate uterus D. uterus didelphys The so called psammoma bodies are found in: A. serous cystadenoma B. mucinous cysadenoma C. clear cell carcinoma D. mucinous cystadenoma Abnormal uterine bleeding during the pubertal and perimenarcheal period is due to:

150.

151.

152.

153.

154.

12

A. B. C. D.

polycystic ovaries failing ovarian function delayed,aynchronous hypothalamic maturation exogenous estrogen stimulation

155.

Most common cause of vaginal bleeding among postmenopausal women: A. endometrial carcinoma B. endometrial hyperplasia C. endometrial polyp D. atrophic endometrium A woman with postmenopausal bleeding warrants: A. an endometrial biopsy B. observation and Paps smear yearly C. total hysterectomy D. total hysterectomy with bilateral salpingooophorectomy In Meigs syndrome, the ovarian newgrowth is a: A. Fibroma B. mature teratoma C. immature teratoma D. cystadenoma Drug of choice for pregnant patients with Chlamydia infection: A. Doxycycline B. azythromicin C. erythromycin D. ampicilin The so called tobacco pouch appearance of the fallopian tube is seen in: A. gonorrhea infection with tubo ovarian abscess B. Chlamydia infenction with tubo ovarian abscess C. Pelvic tuberculos D. Old tubal ectopic gestation Type of endometrial hyperplasis which is mot likely to progress to endometrial carcinoma:

156.

157.

158.

159.

160.

A.
B. C. D. 161.

simple hyperplasis with atypia cystic hyperplasia with atypia complex hyperplasia without atypia complex hyperplasia with atypia

A post coital test is best done: A. pre menstrual B. right after menses C. periovulatory period D. anytime during the cycle Hysterosalpingography is best done: A. periovulatory period B. before menses C. after menses D. anytime during the cycle Which of the following Paps smear will definitely require colposcopic examination of the cervix: A. AGUS B. ASCUS C. LSIL D. HSIL Which of the following is considered a precursor of cervical carcinoma: A. Metaplasia B. dysplasia C. eversion D. severe eroisions Endometrial carcinoma is LEAST likely if endometrial thickness is: A. 5 mm B. <5mm C. 10mm D. <10mm Cystocele and rectocele occur because of weakness of the (ANS: C page 44) A. uterosacral ligaments B. anal sphincter C. endopelvic fascia D. cardinal ligament The sequence of events leading to menstruation (ANS: C page 106) A. coiling of the arteries, vasoconstriction, decrease in endometrial thickness, vasodilatation, menses B. coiling of the arteries, vasodilatation, vasoconstriction, decrease in endometrial thickness, , menses C. decrease in endometrial thickness, coiling of the arteries, vasoconstriction, vasodilatation, menses D. vasoconstriction, decrease in endometrial thickness, coiling of the arteries vasodilatation, menses Thromboxane differs from prostacyclin in that it (ANS: A page 89-90)

162.

163.

164.

165.

166.

167.

168.

13

A. B. C. D.

causes vasoconstriction causes platelet aggregation is not formed from arachidonic acid is not an eicosanoid

169.

A main action prostaglandin 2 (PGF2) is ( ANS: C page 89-90) A. vasodilatation B. platelet aggregation C. bronchoconstriction D. smooth muscle relaxation A Pap smear is likely to identify all the following except (ANS: B page 150-151) A. cervical squamous cell carcinoma B. gonorrhea C. human papilloma virus D. inflammatory changes Examination of a 3-year-old reveals labial adhesion. The child is able to void without difficulty. One should initially recommend (ANS: A page 278) A. topical estrogen B. surgical separation C. a work-up for sexual abuse D. manual separation in the clinic The major mechanism of DMPA, which accounts for its contraceptive effect, is the (ANS: A page 327) A. inhibition of the midcycle gonadotropin surge B. production of unfavorable endometrial environment C. alteration of tubal motility D. alteration of cervical mucus The most commonly encountered cancer of the breast (ANS: D page 383) A. lobular carcinoma in situ B. lobular infiltrating carcinoma C. ductal carcinoma in situ D. ductal infiltrating carcinoma Fibrocystic breast change is characterized by (ANS: B page 364) A. cyclic enlargement of the lymph nodes B. diffuse bilateral findings C. blunted response to cyclic ovarian hormones D. uniform histologic changes A 52-year-old woman has persistent, unilateral, spontaneous bloody nipple discharge and a cluster of microcalcifications identified by xeroradiography to be 3 cm deep under the nipple of the left breast. The next step in her management should be (ANS: B page 429-430) A. needle aspiration under ultrasound guidance B. repeat mammography in 3 months C. submission of the bloody discharge for cytologic examination D. open biopsy of the left breast on an out patient basis The greatest lifetime risk of breast cancer is associated with a(an) (ANS: C page 430) A. early menarche B. late menopause C. history of oral contraceptive use longer than 10 years D. history of postmenopausal estrogen use longer than 10 years Anitiphospholipid antibodies have been found in the circulation of women with (ANS: A page 425-426) A. recurrent abortions B. systemic lupus erythematosus only C. a history of thrombocytopenia D. a false negative result for syphilis The most consistent symptom of ectopic pregnancy is (ANS: D page 456) A. amoenorrhea B. vaginal bleeding C. subjective symptoms of pregnancy D. abdominal pain A morbidly obese woman undergoes preoperative evaluation for adenocarcinoma of the endometrium. Because of the high risk associated with an abdominal procedure, vaginal surgery is being considered. What tumor marker may be of help in her condition? (ANS D page 934-935) A. human chorionic gonadotropin (hCG) B. carcinoembryonic antigen (CEA) C. alpha-fetoprotein D. CA-125 The cell origin of the most common type of ovarian neoplasm is (ANS B page 958) A. germ cells B. epithelial cells C. stromal cells D. lipoid cells The most common cause of direct maternal deaths in the Philippines: (ANS: A p.4) A. Hemorrhage B. Hypertension C. Infection D. Heart disease Most perinatal deaths are due to the following maternal factors: (ANS: A p.8)

170.

171.

172.

173.

174.

175.

176.

177.

178.

179.

180.

181.

182.

14

183.

A. Premature labor B. Anoxia from maternal hemorrhage C. Postmaturity D. Cephalopelvic disproportion The placental circulation (feto-maternal) is established by this day after fertilization (ANS: D p.57)

A. B. C. D.
184.

12th day 14-15th day 10th day 17th day

The greatest concentration of hCG in human plasma/urine is found in this age of gestation: (ANS: A p.63) A. 8-10 weeks B. 12-20 weeks C. 5-6 weeks D. 20-36 weeks This maneuver of Leopold is often called the umbilical grip and answers the question, What side is the fetal back? (ANS: B p.122) A. first maneuver B. second maneuver C. third maneuver D. fourth maneuver Triple screen as a diagnostic tool for the detection fof Downs syndrome utilizes the following except: (ANS: D p.153) A. unconjugated estriol B. alphafetoprotein C. human chorionic gonadotrophin D. human placental lactogen The earliest access to the prenatal diagnosis of heritable conditions is provided for by this technique: (ANS: B p.154) A. amniocentesis B. chorion villus sampling C. cordocentesis D. MRI Relation of the long axis of the fetus to the long axis of the mother; (ANS: C p.183) A. attitude B. presentation C. lie D. position A 28 y/o at 32 weeks age of gestation consults at a clinic. She had one set of twins both are alive and she had an abortion. Her OB code is: A. G3P2(2102) B. G3P1(1012) C. G3P2 (2012) D. G2P1(1012) The most common cause of arrest disorders in labor is: A. CPD B. Hypotonic uterine dysfunction C. Malposition D. Excessive sedation/anesthesia What is the maneuver were the index and middle fingers are applied over the maxilla in order to free the head? A. Bracht B. Zavanelli C. Pinard D. Mauriceau The following anti-TB drugs can be given in pregnancy except: A. rifampicim B. pyrazinamide C. ethambutol D. isoniazid The most common cause of direct maternal deaths in the Philipppines is: (p. 4) A. Hemorrhage B. Hypertension C. Infection D. Heart Disease Glucose transfer from the mother to the fetus is through: (p.110) Facilitated diffusion Simple diffusion Carrier system Pinocytosis The fern formation of dried cervical mucus is due to the effect of: (p.116) Progesterone Estrogen Human placental lactogen Relaxin Quickening refers to: (p.114) Actual movements of the fetus seen sonologically Perception of first movement by the mother

185.

186.

187.

188.

189.

190.

191.

192.

193.

194. A. B. C. D. 195. A. B. C. D. 196. A. B.

15

C. D. 197.

First documentation that the fetus is alive Rate of fetal movement When is the earliest time that can hCG be detected in maternal urine after implantation? (p.116) 5-7 days 14-16 days 8- 9 days 18-21 days A woman who has had three consecutive abortions and no other pregnancies is called: (p.119) Nullipara Multipara Primipara Primigravida At what age of gestation should screening for glucose be done in a pregnant woman: (P. 125) 18-20 wks 24-28 wks 30-32 wks 34-36 wks The Biophysical Profile represents all of the following EXCEPT: (p.162) Accurate prediction of fetal well-being Indirect measurement of fetal oxygenation Fetal behavioral activities including fetal tone Status of fetal cardiac function The fetus is described as complete breech presentation as: (p 185) Legs and thighs are flexed Legs and thighs are extended Legs are flexed, thighs are extended Legs are extended, thighs are flexed Cervical softening and ripening is brought about by: (p. 196) Collagen breakdown and rearrangement Decreased hyaluronic acid Increased dermaton sulfate Increased collagen synthesis

A.
B. C. D. 198. A. B. C. D. 199. A. B. C. D. 200. A. B. C. D. 201. A. B. C. D. 202. A. B. C. D.

203. The following characterize uterine changes during phase 2 of parturition: (p. 199) A. The uterus is divided into an actively contracting upper part and a relatively passive lower segment B. The whole muscle contract simultaneously and with equal intensity thus increasing the expulsive force on the fetus C. After each contraction, the myometrium of the upper and lower segment becomes fixed at a shorter length and this is known as retraction D. After delivery, the uterus diminish in size and the placenta buckles because of limited elasticity

204. A. B. C. D. 205. A. B. C. D. 206. A. B. C. D. 207. A. B. C. D.

The most ominous sign of fetal compromise: (p. 237) Early deceleration Late deceleration Variable deceleration Alternating deceleration Failure to maintain temperature regulation in the newborn may lead to one or all of the following: (p. 246) Peripheral vasoconstriction Hypoxia Acidosis All of the above Puerperium lasts for how many weeks: (p. 251) 4 wks 5 wks 6 wks 7 wks What is the average maternal weight loss immediately after delivery? (p. 254) 4 kg 5 kg 6 kg 7 kg

208. The lochia consisting of servical mucus and debris from healing tissues and leucocytes, lighter yellow and creamy in color: (p. 254) A. Rubra B. Alba C. Cervicosa D. Serosa 209. A. B. C. D. 210. A. The advantage/s of home delivery is/are: (p. 259) Emotional support from family Less expensive Less tendency for nosocomial infections All of the above This refers to 3 or more consecutive spontaneous abortions: (p. 274) missed abortion

16

B. C. D. 211. A. B. C. D. 212. A. B. C. D. 213. A. B. C. D. 214. A. B. C. D. 215. A. B. C. D. 216. A. B. C. D. 217. A. B. C. D. 218. A. B. C. D. 219. A. B. C. D. 220. A. B. C. D. 221. A. B. C. D. 222. A. B. C. D. 223. A. B. C. D. 224. A. B. C. D. 225. A. B.

serial abortion habitual abortion consecutive abortion The implantation of a fertilized ovum outside the endometrium lining the uterine cavity is called: (p. 279) Eccyesis Ectropion Endosalpingosis Blighted ovum Most ectopic pregnancies are found in the: (p. 279) Cervix Tube Ovary Omentum The triad of symptoms of ectopic pregnancy are: (p. 282) Amenorrhea, syncope and vaginal bleeding Abdominal pain, vomiting and vaginal bleeding Syncope, abdominal rigidity and amenorrhea Amenorrhea, abdominal pain and vaginal bleeding The mechanism of preterm labor in chorioamnionitis is: (p. 294) Stretching of the uterine wall due to inflammation Increased prostaglandin synthesis Increased neutrophilic infiltrates in the cervix Irritation of the fetal membranes HELLP Syndrome is the pnemonic for: (p. 332) Hypertension, elevated liver enzymes, low protein Hemolysis, elevated liver enzymes, low platelet count Hemoglobinuria, elevated liver proteins Hyeprtensive encephalopathy, low liver perfusion The anticonvulsant of choice control of convulsion secondary to pregnancy-induced hypertension is: (p. 339) Methyldopa Hydralazine Magnesium sulfate Diazepam Toxicity of magnesium sulfate therapy may be monitored using: (p. 339) Urine output Deep tendon reflexes Respiratory rate All of the above The most common cause of maternal morbidity in hypertensive disease of pregnancy is: (p. 342) Cerebrovascular accidents Myocardial infarction Pulmonary edema Postpartum hemorrhage In case of discordant twins resulting from twin to twin transfusion, the smaller twin usually presents with: (p. 348) Severe anemia Growth retardation Oligohydramnios All of the above The fetal mortality rate in multifetal pregnancy is highest in: (p. 348) Monozygotic, dichorionic, diamniotic Monozygotic, monochorionic, diamnionic Monozygotic, monochorionic, monoamnionic Dizygotic, dichorionic, diamnionic A pregnancy is considered portterm if it lasts longer than: (p. 363) 280 days 284 days 290 days 294 days The most common cause of respiratory distress syndrome (RDS) in the preterm neonate is: (p. 395) Hyaline Membrane Disease Bronchopulmonary Dysplasia Meconium Aspiration Pneumothorax What antenatal procedure is used to detect fetal lung maturity? (p. 395-396) Cordocentesis Amniocentesis Chorionic villus sampling X-ray of the fetal lungs In breech presentation, engagement is considered to have occurred when this has passed the pelvic inlet: (p. 434) Biparietal diameter Bitrochanteric diameter Knees Navel Which is considered an indication for internal podalic version: (p. 445) Oblique lie in case of fetal demise Delivery of second of twins

17

C. D.

Fetal distress in cephalic presentation with fully dilated cervix Prolapsed cord with 8 cm dilated cervix, live, cephalic

226. A carefully monitored trial of labor may be undertaken in normal term pregnancies with average infants following a previous Cesarean section performed for the following indications, EXCEPT: (p. 458-459) A. Abruption placenta B. Placenta previa centralis C. Contracted pelvis D. Fetal distress due to prolapsed cord 227. A. B. C. D. 228. A. B. C. D. 229. A. B. C. D. 230. A. B. C. D. 231. A. B. C. Cesarean hysterectomy may be indicated as a lifesaving measure in cases of postpartum hemorrhage due to: (p.462) Retained placenta fragments Abruption placenta Cervical lacerations Uterine atony Postpartum hemorrhage is blood loss during the first 24 hours after delivery in excess of : (p.465) 100 cc 300 cc 500 cc 1,000 cc The following are viral infections known to be related to congenital infection syndromes, EXCEPT: (p. 524-525, 533) Herpes virus Cytomegalovirus Toxoplasmosis Rubella What is the daily minimum requirement for elemental iron in pregnancy? (p. 561) 4-6 mg 6-8 mg 8-10 mg 10-12 mg The principal cause of thrombocytopenia in pregnancy is: (p. 564) Idiopathic Premature rupture of bag of waters Drug-induced from intake of prenatal vitamins Pre-eclampsia, eclampsia A primigravid patient at 28 weeks AOG came in due to hypogastric and lumbosacral pain. On physical examination uterine contraction was noted occurring every 5 to 10 minutes. I.E.: cervix is noted to be 1 cm dilated, 50% effaced, intact bag of water. Which of the following is a -adrenergic receptor stimulant that can be used to inhibit preterm labor in this patient? A. Isoxsuprine HCL B. Magnesium Sulfate C. Indomethacin D. Nifedipine ANS: A (APMC page 421) This is the most critical issue in the management of postterm pregnancies? (page 431) A. Number of pregnancies B. Confirmation of gestational age C. Maternal age D. Estimated fetal weight ANS: B (APMC page 431) In all growth-restricted fetuses, which of the following biometric parameters will be the first to show discrepancy in measurement? (page 438) A. Head circumference B. Femur length C. Abdominal circumference D. Biparietal diameter ANS: C (APMC page 438) In a patient with monozygotic twinning, if cell division occurs about 8 days after fertilization, what type of twinning will develop? A. Monoamnionic, monochorionic twin pregnancy B. Dichorionic, diamnionic twin pregnancy C. Diamnionic, monochorionic twin pregnancy D. Conjoined twin pregnancy ANS: A (APMC page 411) A patient at 38 weeks AOG came in labor. I.E. cervix is 5 cm dilated, 50% effaced, intact BOW, cephalic, station -3. Clinical pelvimetry shows the ischial spine is bilaterally prominent, pelvic sidewalls are convergent and concavity of the sacrum is shallow. With these findings, you suspect the presence of: A. Pelvic inlet contraction B. Midpelvic contraction C. Pelvic outlet contraction D. Pelvis is adequate ANS: B (APMC page 204) When the fetal head is fully flexed, the chin lies in front of the chest and the presenting anteroposterior diameter is suboccipitobregmamtic. What type of presentation is this? A. Vertex (occiput) presentation B. Sinciput Presentation C. Brow presentation D. Face presentation ANS: A (APMC page 211) Which Leopolds maneuver will identify the location of the cephalic prominence?

D.
232.

233.

234.

235.

236.

237.

238.

18

A.

Leopolds maneuver 1 Leopolds maneuver 2 Leopolds maneuver 3 Leopolds maneuver 4 ANS : D (APMC page 216)

B.
C. D. 239.

The cardinal movements of labor were as follows:

A.
B. C. D. 240.

Engagement, descent, flexion, internal rotation, extension, external rotation, expulsion Descent, engagement, flexion, internal rotation, extension, external rotation, expulsion Engagement, descent, extension, external rotation, flexion, internal rotation, expulsion Descent, engagement, extension, external rotation, flexion, internal rotation, expulsion ANS : A (APMC page 242)

A multigravid patient at 39 weeks AOG came in due to hypogastric and lumbosacral pain. Which of the following additional statements will characterize that the patient is in true labor?

A.
B. C. D. 241.

The cervix is dilated and effaced Contraction is stop by sedation The interval in between contraction is longer The intensity of uterine contraction is unchanged ANS: A (APMC page 248)

This type of deceleration is characterized by symmetrical fall in FHR beginning at or after the peak of a uterine contraction and returning to baseline only after contraction has ended. A. Early deceleration

B.
C. D. 242.

Variable deceleration Late deceleration Sinusoidal pattern ANS: C (APMC page 268

This is considered as the most crucial event for the further development of an antral follicle A. activation of the aromatase system by FSH B. activation of the aromatase system by LH C. luteinization of the granulosa cell D. luteolysis ANS: A (APMC page 28) Rubins pathologic criteria for cervical pregnancy include the following, EXCEPT: A. there must be cervical glands opposite the placental attachment B. he attachment of the placenta to the cervix must be intimate C. fetal elements must be present in the corpus uteri D. the whole portion of the placenta must be situated below the entrance of the uterine vessels ANS: B (APMC page 333) A 28-year old primigravida comes to your clinic on her 30th week of gestation complaining of chest pain. Which of the following findings will strengthen the diagnosis of heart disease in pregnancy? A. an increase in the area of cardiac dullness B. The left cardiac wall is displaced to the left by about inch C. Cardiomegaly D. ECG findings of slight axis deviation, occasional T waves and lowering of the waves ANS: C (APMC page 107) The increase in plasma volume during pregnancy by about 50 to 65% reaches its peak during the:

243.

244.

245.

A.
B.

1st trimester Midtrimester 3rd trimester During labor ANS: B (APMC page 107)

C.
D.

246.

Nayda, an 18-year old primigravida comes to the clinic on her 12th week of gestation for her 1st check-up. She is curious as to when she can start to perceive fetal movements. You will reassure her that this will occur : A. anytime now

B. C. D.
247.

between the 16th and 18th week between the 18th and 20th week between the 20th and 24th week ANS: C (APMC page 127)

Which of the following immunizations is recommended for routine use during pregnancy? A. Rubella B. Tetanus toxoid C. C. Varicella-zoster D. D. Hepatitis A ANS : B (APMC page 197) In vertex presentation, the posterior fontanel is lowermost and the presenting head diameter is the: A. Occipitofrontal B. Biparietal C. Suboccipitobregmatic D. submentobregmatic ANS: C (APMC page 211) This movement of the head refers to turning of the occiput from its original transverse oblique position towards the symphysis pubis or less commonly towards the hollow of the sacrum: A. internal rotation B. extension

248.

249.

19

C. D. 250.

external rotation flexion ANS: A (APMC page 244)

Overt diabetes during pregnancy is diagnosed by: A. random plasma glucose level > 126 mg/dl B. polydipsia, polyuria or unexplained weight gain C. fasting glucose of 105-125 mg/dl

D.
251.

fasting plasma glucose of 126 mg/dl or higher ANS: D (APMC page 591)

Among mothers with thyrotoxicosis. Their neonates are usually A. euthyroid

B. C.
D. 252.

With goitrous thyrotoxicosis with nonimmune hydrops with hypothyroidism ANS: A (APMC page 596)

Subclinical hypothyroidism is diagnosed when there is: A. low serum thyrotropin level and low thyroxine level

B.
C.

high serum thyrotropin level and normal thyroxine high serum thyrotropin level and high thyroxine level low serum thyroxine level and normal thyroxine level ANS: B (APMC page 596)

D.
253.

A 38 year old G4P3 at 18 weeks age of gestation has BP of 150/100 mmHg with no other signs and symptoms. Her prepregnancy BP was 130-140 / 80-90 mmHg controlled by intake of a calcium channel blocker. Urinalysis is negative for proteinuria. The most probable diagnosis of this patient: A. gestational hypertension B. pre-eclampsia C. chronic hypertension D. chronic hypetension w/ supper imposed pre eclampsia ANS: C (APMC page 393) A 28 year old G1P0 at 35 weeks AOG consulted in the ER with chief complaint of severe abdominal pain. Accompanied by moderate vaginal bleeding. Her BP is 180/120 mmHg, PR-88/min, RR: 22/min. On palpation of the abdomen the uterus was tetanically contracted and the FHT was faint. The most probable diagnosis is: A. placenta previa B. abruptio placenta

254.

C.
D. 255.

acute abdomen uterine rupture ANS: B (APMC page 374 & 376)

Pneumococcal vaccine recommended for A. all pregnant patients B. immunocompromised patients C. pregnant patients with influenza pneumonia D. hypertensive pregnant patients ANS: B (APMC page 586) A G3P2 PU at 12 weeks is complaining of cough, weight loss and hemoptysis. The most important screening test at this time is A. chest X-ray with abdominal shield B. tuberculin test C. sputum AFB smear D. sputum culture for TB ANS: B (APMC page 586) Among the anti-TB regimen, the drug which is contraindicated during pregnancy is A. INH

256.

257.

B. C.
D. 258.

Rifampicin Ethambutal Streptomycin ANS: D (APMC page 586)

A 30 year old G3P2 at 24 weeks AOG complained of urinary frequency, dysuria, urgency, fever, chills and costovertebral angle tenderness. Urinalysis showed pyuria and bacteriuria. The most probable diagnosis is A. asymptomatic bacteriuria B. cystitis C. urethritis D. acute pyelonephritis ANS: D (APMC page 640-641) This type of forceps has a double pelvic curve to facilitate application to the after coming head in breech presentation A. Simpson forceps

259.

B.
C. D. 260.

Barton forceps Piper forceps Kielland

ANS : C (APMC page 529) The following are contraindications for vacuum extraction EXCEPT

A. B. C.
D.

fetal postmaturity prior fetal scalp trauma active bleeding nonvertex presentation ANS: A (APMC page 533)

20

261.

This is known as the low longitudinal incision of the uterus, used to delivery babies abdominally A. Classical cesarean section

B.
C. D. 262.

Kerr incision Kronig incision a and c ANS: C (APMC page 541)

A 25 y/o G2P2, three weeks postpartum came in because of profuse vaginal bleeding. She was afebrile with a BP of 100/70 mmHg and pulse rate of 105/min. What is the most likely diagnosis? A. subinvolution

B.
C. D. 263.

wound infection post-partum ovarian vein thrombosis necrotizing faciitis ANS: A (APMC page 560)

What is the first histologic indication of the effect of progesterone on the endometrium? A. presence of mitotic figures B. glandular proliferation C. subnuclear vacuolization D. stromal edema ANS: C (APMC page 44) At this gestational age, gross examination of the external genitalia identifies the sex of the fetus: A. 8 weeks B. 11 weeks C. 12 weeks D. 16 weeks ANS: D (APMC page 81) The most specific test for HIV is A. Enzyme-linked Immunosorbent Assay

264.

265.

B.
C. D.

Polymerase Chain Reaction Western Blot Technique Viral Culture ANS: B (compre page 686)

266.

A 26 y/o patient was seen in the OPD complaining of a vaginal discharge. The vaginal pH was 4.5. What is the most likely cause of the vaginitis A. Bacterial Vaginosis B. Trichomoniasis C. Atrophic Vaginal Discharge D. Moniliasis ANS: D (compre page 669) Infections with this virus is strongly linked to CIN : A. HSV II B. CMV C. HPV D. HIV ANS: C (compre page 868) The least of the nodal involvement for distant metastasis of cervical cancer is via the: A. Paracervical nodes

267.

268.

B.
C. D. 269.

Hypogastric nodes External iliac nodes Presacral nodes ANS: D (compre page 898)

A 45 year old G2P2 female came in due to postcoital bleeding. Pelvic exam revealed a nodular cervix which bleeds to touch, sidewalls are free of lesion. What is the stage of the disease? A. Carcinoma-in-situ B. Stage IB

C.
D. 270.

Stage IIA Stage IIB ANS: B (compre page 899)

Operative treatment for cervical cancer that guarantees the removal of the entire cervix and uterus with the ureters undisturbed.

A.
B. C. D. 271.

Class I Class II Class IV Class V ANS; A (compre page 902)

A 54 year old G5P5 woman came in due to metrorrhagia. Shes of heavy build, and a smoker of the following risk factors, what would have the least risk for developing endometrial carcinoma? A. Smoking B. Obesity C. Multiparity D. late menopause ANS: C (compre page 920) A tumor metastatic to the ovary, usually bilateral, consisting of signet ring cells, that usually originate from GIT is A. mucinous tumor B. krukenberg tumor

272.

C.

serous tumor

21

D.
273.

sex-cord stromal tumor ANS: B (compre page 956)

A 30 year-old G0P0 was found to have stage 1a grade 1 serous cystadenocarcinoma of the ovary. The best form of management is: A. TAHBSO B. TAHUSO

C.
D. 274.

USO with follow up Ultrasound of the contralateral ovary USO with wedge resection of contralateral ovary ANS: C (compre apge 974)

A 13 year old who has not had menarche was brought in for consult with the following features: short stature, low breast development, with webbing of the neck. The diagnosis is: A. Gonadal agenesis B. Turner syndrome C. Hypothalamic failure D. Testicular feminization ANS: B (compre page 1105) It is a malformation of a 46-XX individual with a normal ovarian function resulting in failure of the uterus and vagina to form A. Androgen resistance syndrome

275.

B.
C. D. 276.

Rokitansky Kusterhaus syndrome Gonadotropin resistance ovary syndrome Kallmann syndrome ANS: B (compre page 1107)

A 35- year old female came in for infertility work-up. When is the best time to do hysterosalpingography (HSG)? A. during menses

B.
C. D. 277.

during the week following the end of menses at mid cycle before menses ANS: B (compre page 1176)

What is the pathognomonic symptoms of menopause caused by estrogen depletion? A. hot flushes B. osteoporosis C. Alzheimer-like symptoms D. irritability ANS: A (compre page 1224) Which of the following statements is true regarding LH? A. LH acts on the theca cells to produce androgen. B. LH surge inhibits ovulation. C. LH stimulate follicular growth. D. LH receptors are found in the pituitary. Answer: a Reference: Textbook of Obstetrics 1st edition, Baja-Panlilio et al, page 23

278.

279.

What event induces follicular rupture and subsequent ovulation? A. FSH surge B. LH surge C. Increase in progesterone levels D. Decrease in progesterone levels Answer: b Reference: Textbook of Obstetrics 1st edition, Baja-Panlilio et al, page 30

280.

Which of the following explains uterine enlargement during pregnancy? A. It is due to increased number of muscle fibers. B. It is due to decreased number of fibrous tissue. C. It is due to increased synthesis of Polysaccharide. D. Muscle cells hypertrophy to 10 times the non pregnant state. Answer: a Reference: Textbook of Obstetrics 1st edition, Baja-Panlilio et al, page 96

281.

Which of the following are endometrial changes during pregnancy? There is conversion of secretory endometrium into decidua. Stromal cells are diffused and far apart. Stromal cells are anucleic and avascular. There is a decrease in the endometrial glands. Answer: a Reference: Textbook of Obstetrics 1st edition, Baja-Panlilio et al, page 96

282.

Which of the following are cervical changes during pregnancy? There is a decrease in vascularity. The cervix is edematous. There is a decrease in glands. Softening of the cervix starts at 10 wk AOG. Answer: b Reference: Textbook of Obstetrics 1st edition, Baja-Panlilio et al, page 102

283.

Which of the following are cardiovascular changes in pregnancy? There is an increase in heart beat to 10 beats per minute. There is a decrease in cardiac output.

22

There is marked fall in plasma volume before delivery. There is an increase in diastolic pressure. Answer: a Reference: Textbook of Obstetrics 1st edition, Baja-Panlilio et al, page 104 284. The following are respiratory changes in pregnancy except: Vital capacity is increased. Respiratory rate is increased. Tidal volume is decreased. Respiratory rate volume is decreased. Answer: c Reference: Textbook of Obstetrics 1st edition, Baja-Panlilio et al, page 105 285. What is the average increase in maternal blood volume during pregnancy? A. 10 % B. 25 % C. 50 % D. 75 % Answer: c Reference: Textbook of Obstetrics 1st edition, Baja-Panlilio et al, page 104 286. Which of the following is a positive sign of pregnancy? A. Amenorrhea B. Ballottement C. Recognition of the fetus by ultrasound D. Enlargement of the abdomen Answer: c Reference: Textbook of Obstetrics 1st edition, Baja-Panlilio et al, page 113 287. A radiologic evidence of fetal death that shows significant overlapping of fetal skull bones A. Spaldings sign B. Roberts sign

C.
D.

Brakemans sign Ladins sign

Answer: a Reference: Textbook of Obstetrics 1st edition, Baja-Panlilio et al, page 118 288. The mean duration of a normal pregnancy calculated from the first day of LMP averages close to . A. 40 weeks B. 37 weeks

C.
D.

42 weeks 28 weeks

Answer: a Reference: Textbook of Obstetrics 1st edition, Baja-Panlilio et al, page 120 289. At what age of gestation can fetal cardiac activity be detected by ultrasound? A. 4 weeks ovulation age B. 6 weeks ovulation age C. 8 weeks gestational age D. 10 weeks gestational age Answer: a Reference: Textbook of Obstetrics 1st edition, Baja-Panlilio et al, page 75 290. At what age of gestation will the testes start to descend? A. 24 weeks B. 28 weeks C. 32 weeks D. 36 weeks Answer: c Reference: Textbook of Obstetrics 1st edition, Baja-Panlilio et al, page 78 291. What is the term used to describe the encirclement of the largest fetal head diameter by the vulvar ring? A. full dilatation B. full effacement C. crowning D. engagement Answer: c Reference: Textbook of Obstetrics 1st edition, Baja-Panlilio et al, page 226 What is the main disadvantage of median episiotomy? A. Pain B. difficult repair

292.

C.
D.

rectal extension more blood loss

Answer: c Reference: Textbook of Obstetrics 1st edition, Baja-Panlilio et al, page 229 293. What is the purpose of the WHO partograph? A. To show accurately the course of labor. B. To identify patients needing timely intervention. C. To identify neonatal outcome.

23

D.

To predict a parturients complications.

Answer: b Reference: Textbook of Obstetrics 1st edition, Baja-Panlilio et al, page 224 294. A 31 year old, primigravid was admitted 5 cms. station -1 at 12 noon. In an hour she was 7cm, station 0. At 2 pm she was 9 cm Station +1 and was fully dialted at 3 pm. She delivered shortly thereafter. What is your assessment? A. normal labor B. precipitate labor C. protracted descent D. protracted active phase dilatation Answer: a Reference: Textbook of Obstetrics 1st edition, Baja-Panlilio et al, page 224 A 32 year old G6P5 patient comes in 7cm. dilated at 10 am. What time do you expect her to be fully dilated if labor progress is normal? A. 11 am B. 12 am C. 1 pm D. 2 pm Answer: a Reference: Textbook of Obstetrics 1st edition, Baja-Panlilio et al, page 224

295.

296.

A G3P2 patient had a cesarean delivery because of a previous myomectomy. In the ER she was 3-4 cms, breech presentation, with contractions every 4-7 minutes. What is the best management for this patient? A. Await vaginal delivery. B. Start oxytocin to improve contractions. C. Have her prepared for an emergency cesarean delivery. D. Send her home but advise to return with regular contractions. Answer: c Reference: Textbook of Obstetrics 1st edition, Baja-Panlilio et al, page 433-458

297.

A 21 y/o primigravid presents with severe abdominal pain associated with shoulder pain and dizziness. BP 90/60, PR 110/min. These is rebound tenderness on examination of the abdomen and exquisite tenderness on wiggling the cervix. The patient has history of completion of abortion by curettage at 6 weeks gestation 2 weeks prior to consult. Histopath findings reveal decidual reaction. Pregnancy test is (+). The most likely diagnosis is . A. Unruptured ectopic pregnancy

B.
C. D.

Ruptured ectopic pregnancy Threatened uterine abortion Ruptured corpus luteum

Answer: b Reference: Textbook of Obstetrics 1st edition, Baja-Panlilio et al, page 282 298. A 39 y/o, G4P3, 33 weeks gestation presents with minimal bleeding. Physical examination is unremarkable. Ultrasound report is total placenta previa. The most appropriate treatment is . A. Cesarean delivery B. Vaginal delivery C. Double set-up examination

D. Expectant management Answer: d Reference: Textbook of Obstetrics 1st edition, Baja-Panlilio et al, page 314
299. A 22 year old G2P1 (1001) at 35 weeks AOG has a chief complaint of regular uterine contractions for 12 hours. She has stable vital signs. Fundic height is 27 cm, estimated fetal weight is 2,400 grams. Fetal heart tones are 148 bpm and presentation is cephalic. On internal examination, the cervix is 7 cm dilated, fetal head at station -1, intact bag of waters. What would be your management? A. Start IV tocolytics to control labor. B. Send her home with prescription for oral tocolytics. C. Administer betamethasone. D. Admit her and allow labor to progress. Answer: d Reference: Textbook of Obstetrics 1st edition, Baja-Panlilio et al, page 358 What is the ACOG recommendation in managing a postterm pregnancy with a favorable cervix? A. Immediate caesarean section B. Induction of labor C. Continue antenatal surveillance with NST and AFI D. Continue antenatal surveillance with CST Answer: b Reference: Textbook of Obstetrics 1st edition, Baja-Panlilio et al, page 364 The following finding with postterm pregnancy mandates delivery. A. active fetal movement B. oligohydramnios C. accelerations on fetal tracings D. non-reactive NST Answer: b Reference: Textbook of Obstetrics 1st edition, Baja-Panlilio et al, page 364 A 17 y/o. primigravid registers at 16 weeks gestation with a BP of 100/60. At 38 weeks gestation, she is seen in the clinic with a BP of 146/94 and negative proteinuria. She is admitted in the hospital for further evaluation, where, on overnight observation, she has persistent BP above 140/90. A 24-hour urine protein determination is 20mg. What is your diagnosis? A. Gestational HPN B. Mild Preeclampsia C. Severe Preeclampsia D. Chronic HPN

300.

301.

302.

24

Answer: a Reference: Textbook of Obstetrics 1st edition, Baja-Panlilio et al, page 331 303. What is the role of the hepatitis B surface antigen (+) patient in the genesis of hepatitis B infection? A. Chronic carrier state B. Chronic active hepatitis

C.
D.

Acute hepatitis B Facilitates entry of Hepatitis A virus

Answer: a Reference: Textbook of Obstetrics 1st edition, Baja-Panlilio et al, page 529 304. What is the most common mode of transmission of Hepatitis B? A. Through IV drug use

B.
C. D.

Eating of contaminated food Anal intercourse Mother-to-infant route

Answer: d Reference: Textbook of Obstetrics 1st edition, Baja-Panlilio et al, page 529 305. A 21 y/o G1P0 12rh week AOG complained of sore throat, cough and fever and developed lymphadenopathy. What is your main consideration? A. Varicella zoster post-auricular

B.
C. D.

Rubella Rubeola Measles

Answer: b Reference: Textbook of Obstetrics 1st edition, Baja-Panlilio et al, page 576 306. A primigravid at 39 weeks delivered by low forceps extraction. On the first post-partum day, the patient complained of excruciating vulvar pain. On examination, there was a fluctuant, violaceous gray, 8x6 cm mass on the postero-lateral aspect of the vulva, tense and tender on palpation. What is your diagnosis? A. Gartners duct cyst

B.
C. D.

Bartholins glands abscess Bartholins cyst Vulvar hematoma

Answer: d Reference: Textbook of Obstetrics 1st edition, Baja-Panlilio et al, page 480

307.

R.C., 28 y/o, G2P2, delivered spontaneously to a live fetus 8 days ago at home assisted by a nurse. She developed high-grade fever on the 5th post-partum day associated with body malaise, lower abdominal pain & moderate vaginal bleeding. On admission, the patient was febrile and on IE, the cervix was open, the uterus was enlarged to 18 week-size & both adnexae were tender and foul-smelling lochia was noted. Your impression is . A. Puerperal infection B. Pelvic inflammatory disease

C.
D.

Uterine subinvolution Retained placenta fragments

Answer: a Reference: Textbook of Obstetrics 1st edition, Baja-Panlilio et al, page 472

308.

You are attending to a 25 y/o, G3P3 who just delivered vaginally to a live term fetus. Five minutes later, signs of placental separation appeared followed by profuse vaginal bleeding. Bleeding before placental delivery is called . A. Early postpartum hemorrhage B. Late postpartum hemorrhage C. Third-stage bleeding D. Puerperal bleeding Answer: c Reference: Textbook of Obstetrics 1st edition, Baja-Panlilio et al, page 467

309.

The definitive treatment in a 40 y/o, G8P5 (5,0,3,5) who developed septic shock due to endomyometritis is . A. Surgical removal of the nidus of infection B. Dilatation and curettage C. Triple antibiotic therapy D. D & C after antibiotic coverage Answer: a Reference: Textbook of Obstetrics 1st edition, Baja-Panlilio et al, page 473-474 310. A 16 y/o girl is complaining of irregular menstruation.What would be the ideal treatment to menstrual cycle? A. Oral contraceptive pills B. NSAIDs C. Cyclic estrogen D. Cyclic progesterone Answer: d Reference: Comprehensive Gynecology 3rd edition, page 1032 311. When is it recommended to start doing Pap smear for screening of cervical cancer? A. At age 25 years old B. At age 21 years old C. At age 20 years old D. At age 18 or earlier when sexually active Answer: d Reference: Comprehensive Gynecology 3rd Edition, page 817 restore her

25

312.

Which of the following increases the risk of developing endometrial cancer? A. Multiparity

B. C.
D.

Use of contraception pills Polycystic ovarian symdrome Menopause at age 40

Answer: c Reference: Comprehensive Gynecology 3rd Edition, pages 866-867 313. What is the most appropriate treatment for a 20 year nulligravid diagnosed with simple hyperplasia without atypia by curettage? A. Endometrial oblation

B.
C. D.

Continous or cyclic progestin Total hysterectomy External radiotherapy

Answer: b Reference: Comprehensive Gynecology 3rd Edition, page 871 314. Which of the following is classified as homologous type of sarcoma? A. High grade endometrial stromal sarcoma

B.
C. D.

Liposarcoma Chondrosarcoma Perabdo myosarcoma

Answer: a Reference: Comprehensive Gynecology 3rd Edition, page 888 315. What type of ovarian tumor is Brenner tumor classified? A. Epithelial tumor B. Stromal tumor C. Germ cell tumor D. Sex cord tumor Answer: a Reference: Comprehensive Gynecology 3rd Edition, page 904 A patient underwent THBSO omentectomy and lymph node dissection for an ovarian cancer. Histopathological report showed the tumor in the right ovary measured 10x8x8 cm ruptured with extension to the fallopian tube and omentum. One para aortic node showed malignant cells. What is the stage? A. Stage II-C

316.

B.
C. D.

Stage III-A Stage III-B Stage III-C

Answer: d Reference: Comprehensive Gynecology 3rd Edition, page 912 317. What is the primary treatment of malignant epithelial ovarian tumor? A. Chemotherapy B. Surgical removal of the tumor C. Radiotherapy D. Chemo and radiotherapy Answer: b Reference: Comprehensive Gynecology 3rd Edition, page 918

318.

Definitive diagnosis of vulvar carcinoma is established by A. Toluidine blue test

B.
C. D.

Colposcopy of the vulva Pap smear of the vulva Biopsy

Answer: d Reference: Comprehensive Gynecology 3rd Edition, page 949

319.

The major symptom of most vulvar atypias is . A. Bleeding B. Pain C. Pruritus D. Burning sensation Answer: c Reference: Comprehensive Gynecology 3rd Edition, page 948 A 3 y/o girl was brought to the Emergency Room because of vaginal bleeding. On inspection of the external genitalia, there was a mass protruding through the vaginal introitus. It measured 4x3 cm and resembled a cluster of grapes forming multiple polypoid masses. The most likely diagnosis is . A. Vaginal adenosis B. Sarcoma botryoides C. Epidermoid cancer D. Condyloma acuminata Answer: b Reference: Comprehensive Gynecology 3rd Edition, page 981 The most common symptom of vaginal cancer is .

320.

321.

A.
B.

C.

D. Answer: b Reference: Comprehensive Gynecology 3rd Edition, page 975

Pain abnormal bleeding urinary frequency tenesmus

26

322.

Which of the following patients with PID must be hospitalized? A. A 22 y/o G2P2 with adnexal tenderness B. A 21 y/o G1P0 at 10 weeks AOG C. A 35 y/o G4P4 with fever D. A 38 y/o G1P1 who is an OCW Answer: b Reference: Comprehensive Gynecology 3rd Edition, page 677 What is the most common route of spread of the organisms in PID? A. Lymphatic spread

323.

B.
C. D.

Hematogenous route Direct extension Ascending route

Answer: d Reference: Comprehensive Gynecology 3rd Edition, page 663 324. What is the route of spread of abdomino-pelvic Kochs? A. Direct extension B. Lymphatic spread C. Hematogenous spread D. ascending route Answer: c Reference: Comprehensive Gynecology 3rd Edition, page 684 On histologic sections, what are the findings highly suggestive of AP Kocks? A. Islands of endometrial glands B. Necrotic foci in the endometrium C. Langhans glant cell D. Absence of stroma Answer: c Reference: Comprehensive Gynecology 3rd Edition, page 684 Which of the following is an indicator of ovulation? A. Progesterone level of 10 mg/ml during luteal phase B. 100F to 110F temperature C. estradiol level 50mg/ml D. Leteinizing hormone level 15mg/ml Answer: a Reference: Comprehensive Gynecology 3rd Edition, page 1118 Which of the following is required in semen analysis? A. 2-3 days abstinence before collection B. collection done by masturbation at home C. use of plastic cup for urine samples D. submission of aspirated semen from vagina after coitus Answer: a Reference: Comprehensive Gynecology 3rd Edition, page 1119 WHO standard for normal semen analysis include . A. 40 M sperm count B. 60% M motility C. 60% M morphology D. 10 M white cell count Answer: a Reference: Comprehensive Gynecology 3rd Edition, page 1119 Which of the following is required for hysterosalpingogram? A. Done post menstrual, pre-ovulatory B. Routine antibiotics given after procedure C. Test for tubal function D. Oil based contrast medium is better than water based medium Answer: a Reference: Comprehensive Gynecology 3rd Edition, page 1121 Which of the following organisms are hostile to sperm in the vagina? A. Ureaplsma urealyticum B. Amoeba

325.

326.

327.

328.

329.

330.

C.
D.

Mycobacterium tuberculosis Clostridia

Answer: a Reference: Comprehensive Gynecology 3rd Edition, page 1124 331. A 12 y/o girl was noted to have breast budding. What is the average age you would expect her to experience menarche? A. 10 year old B. 12 year old C. 14 year old D. 16 year old Answer: c Reference: Comprehensive Gynecology 3rd Edition, page 1046 A 45 y/o patient consulted at your clinic because of amenorrhea of 10 months duration. She would want to know if indeed she is menopausal already. What laboratory test will you request? A. Estrogen assay B. FSH assay C. Progesterone assay

332.

27

D.

Prolactin assay

Answer: b Reference: Comprehensive Gynecology 3rd Edition, page 1161 333. What will be the first index of declining ovarian function? A. Increase in FSH B. Declining inhibin levels C. Increase in LH D. Increase in prolactin Answer: b Reference: Comprehensive Gynecology 3rd Edition, page 1161

334.

Which of the following events is associated with menstrual flow? A. withdrawal of progesterone B. prolonged maintenance of estrogen C. withdrawal of LH D. prolonged maintenance of progesterone MPL 0.25 p: 106 A 47 year old woman has poorly differentiated endometrial carcinoma and a uterine cavity that measures 10 cm in depth. The endocervix has stromal invasion of endometrial carcinoma but no other structure is involved. What is the stage of her disease? A. I A B. I B C. II A D. II B MPL 0.5 p: 929

335.

336.

What endoscopic procedure is utilized to evaluate tubal disease? A. colposcopy B. laparoscopy C. hysteroscopy D. hysterosalpingography MPL 0.5 p: 719 Tuberculosis of the genital tract invariably involves the A. fallopian tubes B. ovaries C. cervix D. vagina MPL 1.0 p: 731 A 26 year old G1P1 had cervical intraepithelial neoplasia involving the entire thickness of the cervical epithelium. What is the best management? A. cryosurgery B. electrocautery C. conization D. hysterectomy MPL 0.5 p: 878 A 25 year old woman and her husband use natural family planning as their method of contraception. Her menstrual cycle length range from 26 to 32 days. She does not measure her basal body temperature. The time of her maximum or peak fertility with the first day of her menses defined as day 1 would be between A. day 1 14 B. day 8 14 C. day 8 21 D. day 14 21 MPL 0.5 p: 301 What is the drug of choice for bacterial vaginosis? A. tetracycline B. fluconazole C. cefoxitin D. metronidazole MPL 1.0 p: 671 What type of myoma insinuates itself between the leaves of the broad ligament? A. parasitic B. serosal C. interstitial D. intraligamentary MPL 0.5 p: 498 What tumor will be positive for the tumor marker HCG? A. embryonal carcinoma B. choriocarcinoma C. endodermal sinus tumor D. adenocarcinoma MPL 0.5 p: 1053

337.

338.

339.

340.

341.

342.

343.

What type of abnormal uterine bleeding is excessive in amount and duration of flow occurring at regular interval? A. Menorrhagia B. metrorrhagia

28

C. D.

polymenorrhea menometrorrhagia MPL 0.5 p: 1079

344.

A 67 year old had episodes of vaginal bleeding 15 years after menopause. Biopsy revealed endometrial hyperplasia. Transvaginal sonogram reveals a solid adnexal mass of 4 cm diameter. She is not on hormonal replacement therapy. What is the most likely cause of the bleeding? A. sertoli-leydig tumor B. granulose cell tumor C. serous cystadenocarcinoma D. endodermal sinus tumor MPL 0.5 p: 173 A 40 year old housewife complains of foul smelling vaginal discharge. On examination, discharge was greenishgray in color, fishy odor, and frothy. What is the primary consideration? A. Yeast B. Trichomoniasis C. Chlamydia D. Bacterial vaginosis MPL 0.5 p: 673 A 36 year old G2P2 has amenorrhea of 11 months with hot flushes. Examination reveals a small uterus. If this is a case of premature ovarian failure, which finding is applicable? A. biphasic basal body temperature B. endometrial hyperplasia C. elevated FSH > 40 IU/ml D. tonically raised LH MPL 0.5 p: 1100 What is a contraindication to hormonal replacement therapy? A. history of pulmonary thromboembolism B. hot flushes, insomnia in a 50 year old with irregular menses C. history of fibrocystic disease of the breast D. elevated serum lipids MPL 0.33 p: 1249 Which of these is the most likely site for implantation of endometriosis? A. omentum B. appendix C. bladder mucosa D. peritoneum of culdesac MPL 0.5 p: 536 Which disease of the vulva is best treated with wide excision? A. hidradenitis suppuritiva B. invasive vulvar cancer C. angioma D. contact dermatitis MPL 0.5 p: 489 What ovarian tumor will most commonly cause precocious puberty ? A. teratoma B. luteoma C. granulosa cell tumor D. sertoli leydig tumor MPL 0.5 p: 173 A woman with this type of pelvis is more prone to urethrocoele because of more force of the fetal head on this area during descent in labor A. platypelloid B. anthropoid C. gynecoid D. android MPL 1.0 p: 571 Stage III vaginal cancer means extension of lesion up to A. vaginal wall B. subvaginal tissue C. pelvic wall D. rectal mucosa MPL 0.33 p:1028 On endometrial biopsy, glycogen-rich subnuclear vacuoles were seen in the base of the cells lining the glands. What does this mean? A. the woman is in her late luteal phase B. ovulation has occurred C. she is progesterone deficient D. menstruation is about to set in MPL 0.33 p: 104 Which theory of endometriosis best explains remote sites of the disease such as in the spinal cord, nasal septum or lungs? A. iatrogenic dissemination B. coelomic metaplasia C. immunologic changes D. lymphovascular metastasis MPL 0.33 p: 533

345.

346.

347.

348.

349.

350.

351.

352.

353.

354.

29

355.

Which is an abnormal semen parameter?

A.
B.

white cell count of 105 per ml sperm morphology 30 % sperm count 40 x 106 per ml progressive motility in 65 % MPL 0.33 p: 1175

C.
D.

356.

A 32 year old has been having her regular pap smear for the last ten years. Her latest result however revealed low grade SIL. What is the next step to do? A. colposcopic examination B. repeat smear in 4 months C. conization D. laser ablation MPL 0.33 p: 873

357.

A 19 year old was brought to the ER because of acute pain. There was also vaginal spotting. Examination revealed a small tender adnexal mass. Based on the LMP, she is on day 25 of her cycle. Pregnancy test was negative. What is the most likely diagnosis? A. ectopic pregnancy B. acute salpingitis C. ruptured corpus luteum D. ruptured endometrial cyst MPL 0.33 p: 509 A 28 year old nulligravida with primary infertility and cyclic pelvic pain was diagnosed to have endometriosis. She was placed on hormonal treatment. However, while on this drug, she developed acne, hirsutism, and deepening of the voice. What is a better alternative? A. laparoscopic fulguration B. GnRH agonist C. Progesterone injectables D. Oral contraceptives MPL 0.33 p. 546

358.

359.

A 35 year old consulted because of vesicular lesions in the vulva. She related that several days prior to the appearance of the vesicles, she felt numbing sensation over her vulvar skin. Which is the best treatment? A. metronidazole B. acyclovir C. doxycycline D. penicillin MPL 0.33 p. 659 360. A 17 year old consulted because of lower abdominal pain of 3 days duration described as constant and dull. On speculum exam, purulent vaginal discharge was noted. Patient was febrile at 38.5 C. Pertinent finding on internal exam: tenderness with motion of the uterus and cervix. What is the most likely etiologic agents? A. Neisseria gonorrheae and Chlamydia trachomatis B. Trichomonas vaginalis and gardnerela C. Anaerobic gram negative bacteria D. Pseudomonas MPL 0.5 p. 724 A woman consulted because of spontaneous milky discharge and missed period of 8 weeks. Pregnancy test was negative. Prolactin levels after appropriate blood collection was 35 ng/ml. What should be the next step in the work-up? A. repeat prolactin in a quiet room B. CT scan of the brain C. Cone view of the sella turcica D. TSH determination MPL 0.33 p. 1133 A patient was diagnosed to have squamous cell cervical cancer. Clinical evaluation revealed that the disease has involved the upper third of the vagina. Parametrial tissues were indurated. However, there was no evidence of pelvic well involvement. What is the best treatment for her? A. radiotherapy B. chemoradiation C. RHBLND D. Chemotherapy MPL 0.33 p. 905 A 19 year old consulted because of primary amenorrhea. On clinical evaluation, the patient was found to have breast development but absent uterus. Which of the following can best help arrive at diagnosis? A. progesterone challenge test B. GnRH level determination C. Karyotyping D. Gondal biopsy MPL 0.33 p. 1106 A 19 year old, primigravida, at 34 weeks gestation comes in for severe headache and visual blurring. Previous blood pressure on prior prenatal check up was at 140/90 mm Hg. Presently, her blood pressure was 170/110 mm Hg. She presented with bipedal edema. There was no uterine contractions. Then she developed generalized tonic-clonic seizures last for about 40 secs. What is most likely diagnosis? A. Chronic hypertension B. Severe pre-eclampsia C. Chronic hypertension with superimposed hypertension D. Eclampsia

361.

362.

363.

364.

30

MPL 0.5 365.

p. 764

Which of the following conditions can be predicted by cervicovaginal fibronectin? A. pre-eclampsia B. feta growth restriction C. preterm labor D. premature rupture of membrane MPL 0.33 p. 937 A G2P1 with 7 weeks missed period presents with one week diagnosis of vaginal bleeding and hypogastric pains. Cervix is 1 cm open with intact membranes. What is the diagnosis? A. Threatened abortion B. Imminent abortion C. Inevitable abortion D. Incomplete abortion MPL 0.33 p. 243 Which is a cause of polyhydramnios? A. renal agenesis B. prematurity C. abruption D. esophageal atresia MPL 0.5 p. 526 A 26 year old G3P2, Previous CS I, on her 29th week age of gestation went to the hospital because of episodes of bright red vaginal bleeding upon waking up without other symptoms. Vital signs were normal with no uterine contraction noted. Leopolds maneuver showed a baby in cephalic presentation, floating, FHT was 150/min. What is the diagnosis? A. placenta previa B. placenta abruption C. Premature labor D. Ruptured uterus MPL 1.0 p. 630 A nullipara was admitted on active labor at 5 cms., ruptured bag of waters, station -2. After 3 hours of good contractions, cervix was 5 cms dilated, station -2. What condition is she in? A. prolonged latent phase of labor B. arrest of cervical dilatation C. arrest of descent D. protracted active phase of labor MPL 0.5 p. 499 A patient diagnosed to have asymptomatic bacteriuria is one who has? A. persistently multiplying bacteria in the urine but no symptoms. B. 100,000 organisms in culture specimen with pyuria C. a sterile urine culture with frequency and urgency. D. microscopic hematuria and colony count of 50,000/ml. MPL 0.25 p. 1253 Pigmentation of the face during pregnancy is considered as A. an indication of increased androgen from a male fetus B. a temporary state rapidly regressing postpartum C. a benign but persistent condition D. an indication for immediate hyroxyquinone application MPL 0.33 p. 1430 A 25 year old consulted for vaginal spotting and sharp pelvic pain. History revealed 6 weeks amenorrhea. Pregnancy test was positive. Physical exam elicited cervical wriggling tenderness with a vague mass and tenderness in the left adnexae. Posterior vaginal fornix as full and bulging. BP was 100/60 mm Hg., PR 102/min. What should be done next to this patient? A. culdocentesis B. emergency transvaginal ultrasound C. blood transfusion D. immediate exploration MPL 0.5 p. 890 A woman on her 32nd week of gestation was admitted for threatened preterm labor. Tocolytic management was initiated with magnesium sulfate. Which of the following needs to be monitored closely while on tocolysis? A. SGOT B. Glucose C. Blood pressure D. Potassium MPL 1.0 p. 714 A 20 year old primigravida at 39 weeks came in due to labor pains. This patient was a diagnosed case of rheumatic heart disease. When should she receive the initial dose of ampicillin-gentamycin? A. about 2-3 hours after admission B. prior to amniotomy C. after the first internal examination D. 30 minutes prior to anticipated delivery MPL 0.25 p. 1189 A puerperal women was rushed to the ER because of profuse bleeding. On examination, a sac-like structure was seen out of the introitus with the placenta still attached. A crater-like depression as noted on abdominal palpation. Which of the following is part of good management? A. immediate removal of the placenta before replacement B. give oxytocin to facilitate replacement

366.

367.

368.

369.

370.

371.

372.

373.

374.

375.

31

C. D.

administer fluids and magnesium sulfate for uterine relaxation do a vaginal hysterectomy MPL 0.33 p. 643

376.

A pregnant women consulted because of itchy reddish wheal-like rashes. The lesions were first seen in the abdomen eventually spreading to the rest of the body. She denied having any allergies to food or drugs. What treatment is appropriate? A. antibiotics B. estrogen cream C. corticosteroids D. lindane MPL 0.33 p. 1025 A pregnant woman on her 1st trimester of pregnancy was exposed to a neighbor with german measles. What is the initial step in the management? A. active immunization B. immunoglobulins C. antiviral therapy D. ask history of past infection MPL 0.33 p. 1469 Immediately after an apparently normal labor and delivery, the mother suddenly manifested with dyspnea and went into convulsions. Blood pressure likewise dropped sharply from 120/80 mm Hg. Cardiopulmonary arrest rapidly ensued. What is the probable cause? A. cerebrovascular accident B. eclampsia C. amniotic fluid embolism D. postpartum cardiomyopathy MPL 0.33 p. 660 A G5P4 pregnant uterine 39 weeks was in active labor for 3 hours. IE showed cervix 7 cms dilated, fetal head at station -1 to 0. Suddenly, fetal heart rate decelerated and maternal blood pressure dropped from 120/80 to 90/60 mm Hg. On doing IE, the presenting part appeared to retract. What is the most probable diagnosis? A. abruption placenta B. spontaneous uterine rupture C. acute cord torsion D. uterine atony MPL 0.5 p. 650 A G2P1 PU 31 weeks came to the clinic with complaints of watery vaginal discharge. After noting some fluid pooling at the culdesac, the clinician requested for an ultrasound. What information can be derived from an ultrasound pertinent to management? A. demonstration of site of bag rupture B. confirmation of fetal maturity C. assessment of amniotic fluid volume D. detection of fetal meconium staining MPL 0.33 p: 526

377.

378.

379.

380.

381.

During clinical pelvimetry, the ischial spines were noted to be prominent, the sidewalls convergent and the sacraosciatic notch is narrow. Of the following, what is the most common consequence of these findings? A. this promotes cord prolapse B. there is increase likelihood for transverse arrest of fetal head C. extreme posterior asynclitism is favored D. production of abnormal presentation is facilitated MPL 0.25 p: 756

382.

Anticipating a large baby, the patients legs were removed from the stirrups and sharply flexed upon her abdomen. What will be the effect of this maneuver? A. increase pelvic dimension B. facilitate bearing down effort C. rotates symphysis pubis toward maternal head D. dislodges the posterior shoulder MPL 0.25 p: 461 A primigravida was brought to the ER by the attending midwife after 10 hours of labor. Apparently she has been bearing down for 2 hours already. Exam showed that the fetal scalp is visible at the introitus without separating the labia. The sagittal suture was at AP diameter. The mother appeared weakened by her efforts to bear down. What is the best management? A. strong fundal pressure B. oulet forceps extraction C. cesarean section D. await vaginal delivery MPL 0.25 p: 487 A 20 year old, primigravid, PU 39 weeks presented at the ER in labor for 3 hours. PE revealed multiple vesicular lesions in the vulva and perieal area. IE showed 3 cm cervical dilatation, 50 % effaced, intact bag of waters, cephalic, station -1. Clinical pelvimetry was adequate. What is the best route of delivery? A. cesarean section B. vacuum extraction C. outlet forceps extraction D. spontaneous vaginal delivery MPL 0.5 p: 1496 A 53 A 29 year old, Gravida 5 Para 4 (4-0-0-4), Pregnancy uterine 37 weeks, complaining of hypogastric pain of 2 hours, came in the emergency room. Her vital signs were normal. Fundic height was 35 cms. Fundus is occupied by a hard, round, ballotable mass, fetal back palpated at the left, as well on the right, another hard, round mass on the hypogastric area, fetal heart tones of 143/min heard at the left periumbical area

383.

384.

385.

32

and 152/min heard at the right lower quadrant. On internal examination, the cervix was 5 cms. Dilated, 1 cm long, ruptured bag of waters, cephalic, station -2. All her previous pregnancies were delivered at home. What is the recommended mode of delivery? A. an outright cesarean section B. a vaginal delivery for both C. a vaginal delivery for the first of twin then a cesarean section for the second D. a vaginal delivery for the first of twin then an internal podalic version for the second of twin MPL 0.25 p: 941

386.

Which of the histological presentation is a characteristic of complete molar pregnancy? A. hydropic degeneration and swelling of the villous stroma B. presence of blood vessels in the swollen villi C. proliferation of trophoblastic epithelium with equal degree D. presence of fetus and amnion MPL 0.25 p: 274

387.

When the fetal breech presents with both legs extended and the hips flexed, it is classified as A. complete breech B. incomplete breech C. frank breech D. footling breech MPL 0.5 p: 567 Procedure where the cephalic presented fetus is converted to a footling breech presentation by grasping the fetal legs and turning the position is called A. external cephalic version B. internal podalic version C. partial breech extraction D. pipers forceps MPL 0.5 p: 583 Which of the following is associated with Abruptio placenta? A. Long cord B. Hypertension C. Cord prolapse D. High levels of B-hCG MPL 1.0 p: 813 Which of the following conditions in NOT an indications for classical cesarean section? A. Myoma occupying the fundus of the uterus B. Invasive carcinoma of the cervix C. Placenta previa with posterior implantation D. Transverse lie of a large fetus MPL 0.5 p: 598 What is considered as the major vestibular glands that lie under the constrictor muscles of the vagina A. Paraurethral glands B. Skenes glands C. Sebaceous glands D. Bartholins glands MPL 0.5 p: 17 A 28 year G1P0 sought her first prenatal check up at 12 weeks gestation. Family history is positive for diabetes mellitus (father). When should she have her GCT? A. 16 20 wks B. 24 28 wks C. 32 36 wks D. at term MPL 0.33 p: 1171

388.

389.

390.

391.

392.

393.

What is the most accurate ultrasonographic parameter for fetal aging in the first trimester ? A. femur length B. biparietal diameter C. head circumference D. crown rump length MPL 0.5 p: 391 What do you call a woman who had 3 pregnancies, all of which were aborted? A. nulligravida B. nullipara C. multipara D. primipara MPL 0.5 p: 207 What is the earliest age of gestation when fetal sex can be identified by gross examination of the external genitalia? A. 6 weeks B. 10 weeks C. 14 weeks D. 20 weeks MPL 0.33 p: 113 What is most common type of the female pelvis? A. android

394.

395.

396.

33

B. C. D.

anthropoid platypelloid gynecoid MPL 1.0

p: 35

397.

When does the 2nd stage of labor end? A. when the cervix is fully dilated B. when the baby is fully delivered C. when the placenta is delivered D. right after episiotomy MPL 0.5 p: 423 How much weight does a average weighted woman gain throughout her pregnancy? A. 10 lbs B. 18 lbs C. 24 lbs D. 30 lbs MPL 0.5 p: 452 On histopathology, a placenta is found to have dense stroma containing round cells with granular and vacuolated cytoplasm with vascular and eccentric nuclei. What is age of this placenta? A. 8 weeks B. 14 weeks C. 24 weeks D. 40 weeks MPL 0.33 p: 612 What CTG finding is indicative of head compression? A. sinusoidal pattern B. absent beat to beat variability C. early deceleration D. late deceleration MPL 0.25 p: 452 A woman on her 41st week of pregnancy had a BPS with the following findings during the 30 minutes testsustained breathing movement of 45 secs, five simultaneous limb and trunk movements, 2 episodes of arm flexion and extension, 3 episodes of fetal heart rate accelerations each for 5 secs with fetal movement, amniotic fluid pocket of 3 cms in perpendicular planes. What should be done for this patients based on the BPS score? A. Do a outright cesarean section B. Induce labor first C. Repeat the BPS on the same day D. Repeat the BPS after a week MPL 0.5 p: 1104 When can a puerperal patient start using a hormonal contraceptive method if does not intend to breast feed? A. 2 - 4 weeks postpartum B. 4 - 6 weeks postpartum C. 6 8 weeks postpartum D. when she starts menstruating again MPL 0.33 p: 412 On performing the thirds Leopolds maneuver, the cephalic prominence is palpated on the left lower quadrant and the fetal heart tones is maximally heard at the right lower quadrant, which of the following can be concluded? A. the head is already engaged B. the vertex is presenting C. the head is extended D. the presenting part is the sinciput MPL 0.33 p: 299 On the 2nd postpartum day, a parturient developed a temperature of 39.0 C. Pertinent PE included breast that were edematous, tender, firm, and nodular. What must be done for this patient? A. give analgesic and breast support B. stop breastfeeding immediately C. give antibiotics D. give bromocriptine for 7 days MPL 0.25 P: 413 On physical examination of a pregnant woman, the clinician detected a 2/6 systolic murmur intensifying during inspiration. What should be done for this patient with regards this finding? A. cardiology referral B. ECG C. No intervention needed D. Echocardiogram MPL 0.5 p: 1168 A 21 year old primigravida in labor for 10 hours presented with a 6 cm cervical dilatation. The bag of waters was ruptured and the presenting fetal head was palpated at station 0. What conclusion regarding the pelvis can be made? A. pelvic inlet is adequate B. pelvic midplane is adequate C. pelvic inlet is inadequate D. pelvic midplane is inadequate MPL 0.5 p: 423 What maneuver will the fetal head undergo right after engagement? A. internal rotation

398.

399.

400.

401.

402.

403.

404.

405.

406.

407.

34

B. C. D. 408.

extension flexion descent MPL 0.5

p: 416 21

What part of the uterus forms the lower uterine segment during labor? A. external cervical os B. cornua C. fundus D. isthmus MPL 0.5 p: 21 What diameter of the pelvic inlet can be assessed clinically? A. Obstetric conjugate B. Diagonal conjugate C. Greatest transverse diameter D. Posterior midsagittal diameter of the inlet MPL 0.33 p: 34 What is the major source of progesterone in early pregnancy? A. placental cytotrophoblast B. placental syncytiotrophoblast C. corpus luteum D. placenta MPL 0.25 p: 124 Which of the following antibiotics are contraindicated during pregnancy? A. metronidazole B. tetracycline C. amoxicillin D. erythromycin MPL 0.5 p: 342

409.

410.

411.

412.

Myrna, a primigravida, on her 32nd week AOG, came in for prenatal check-up. Auscultation of the FHT yields negative result and the ultrasound revealed fetal death in utero. How will you manage her? A. hysterotomy B. wait for spontaneous labor * C. prostaglandins D. curettage MPL 0.5 p: 681 In which part of labor does the descent of the presenting part of the fetus is most marked? A. preparatory division of labor

413.

B.
C.

1st stage of labor pelvic division of labor 3rd stage of labor MPL 0.5 p:422

D.
414.

Which of the following maternal changes are decreased during pregnancy? A. gastric emptying time B. maternal circulating blood volume C. maternal insulin D. resting pulse rate MPL 0.25 p: 140 What conclusions can be made when meconium passage in-utero in a term parturient is observed after amniotomy? A. The GIT of the fetus is still immature B. Can be a sign of fetal hypoxia C. Fetal sympathetic system is stimulated D. Esophageal atresia is a consideration MPL 0.25 pp: 675-676 What is the earliest histological evidence of progesterone action in the endometrium A. glandular mitoses B. pseudostratification of nuclei C. stromal edema D. basal vacuolation MPL 0.25 p: 451 Which is the most biologically potent estrogen in the normal menstrual cycle? A. estrone B. estradiol C. estriol D. androstenedione MPL 0.33 pp: 43-44 If her last menstrual period is last Jan 23, 2007, when is her expected date of confinement? A. April 30, 2007 B. June 26, 2007 C. Oct 23, 2007 D. Oct 30, 2007 MPL 1.0 p: 208-209 Lochia serosa is expected to occur A. immediately after birth

415.

416.

417.

418.

419.

35

B. C. D.

after 3 or 4 days after 10 days after 1 month MPL 0.33 pp: 696-697

420.

Through what nerve is pain of uterine contractions transmitted? A. S1 - S2 B. S4 - S5 C. T7 - T8 D. T10 - L1 MPL 0.25 p: 477 What is the antidote for magnesium toxicity? A. Calcium gluconate B. Calcium channel blocker C. Diazepam D. Midazolam MPL 0.5 p: 789 Which of the following is utilized for medical treatment of ectopic pregnancy? A. Dexamethasone B. Vincristine C. Methotrexate D. Adriamycin MPL 0.25 p: 262 Of the following, which is most associated with low birthweight infants? A. Placenta previa B. Diabetes mellitus C. Maternal height of < 5 ft D. Smoking in pregnancy * MPL 0.25 p: 354 A 5 year old was referred to you at the ER due to complaints of perineal pain. What is the best way to examine the vagina of this child? A. dorsal lithotomy B. knee-chest position C. lying on her mothers lap D. standing with one leg raised MPL 0.25 p: 274 Keanna, 32 y/o sexy dancer and ramp model complained of burning sensation over her vulva after a lingerie fashion show where she wore thongs and stringed nylon bikinis. There was pruritus so she used a newlylaunched vaginal wash. Immediately, the burning sensation became worse but she thought it was normal. The next morning, she found her vulva to be erythematous and inflamed. The redness now reached the peri-anal area. What is the diagnosis? A. Vulva vestibulitis B. Mechanical intertrigo C. Allergic dermatitis D. Fungal vulvo-vaginitis MPL 0.33 p: 487 On cut-section of an ovarian mass in a 26 year old nulligravida, there were ufts of hair, sebum and bony cartilage. What type of ovarian mass is most likely in this case? A. Follicular cyst B. Dysontogenetic cyst C. Cystic teratoma (dermoid) D. Theca lutein cyst MPL 0.33 p: 979 What ulcerative lesion of the genital tract is characterized by the presence of groove sign? A. Granuloma inguinale B. Lymphogranuloma venereum C. Chancroid D. Syphilis MPL 0.33 p: 661 What surgical procedure will prevent a woman from experiencing vaginal coitus? A. Manchester Fothergill B. Colpocleisis C. Vaginal hysterectomy D. Perineorrhaphy MPL 0.25 p: 580 Which is the most common predisposing factor to ectopic pregnancy? A. previous genital infection B. smoking C. contraceptive pills D. douching MPL 0.33 p: 254

421.

422.

423.

424.

425.

426.

427.

428.

429.

36

You might also like